Está en la página 1de 238

FACULTAD DE INGENIERIA CIVIL Y ARQUITECTURA

ESCUELA ACDEMICA PROFESIONAL DE INGENIERA CIVIL

SOLUCIONARIO DEL EXAMEN 2010


Ing. Civil Clifton Paucar y Montenegro - REG. CIP. 45773
ABASTECIMIENTOS DE AGUA Y ALCANTARILLADO - CIC404

RESP.DEL REA HIDRAULICA :


ASIGNATURA

MELGAREJO CONDEZO JOSE


RPTA A.-

i)

Iniciamos el proceso de estimacin calculando la dotacin promedio CATASTRAL, en funcin al consumo promedio cuantificado
de EDIFICACIN Y HABITANTE, as como el consumo promedio en Lt./seg.
35,000

Poblacin total localidad ALFA

Habitantes

ZONA A :

Densidad poblacional promedio :

DATOS

TIPO1( unid.)=

500

Lit./viv./da

Lit./hab./da

Lit./viv./da

Lit./hab./da

Lit./viv./da

Lit./hab./da

230
170
150

950
725
520

210
180
125

1,000
750
550

222
188
166

Mximo
900
Mnimo
700
Promedio
500
TOTAL VIVIENDAS

(RED EXISTENTE)

3.95 habitantes por vivienda


TIPO2( unid.) =

1,330

D12+F12+H12+J12

Consumo promedio por edificacin/ da =

400

TIPO3( unid.) =

250

1,000 x 2,300 885 x 2,020 500 x 1,250 300 x 1,118


(1,000865 500300 )

(D12*C16+F12*E16+H12*G16+) / TOTAL DE VIVIENDAS

Poblacin en el Ao 2009:

15.00%

35,000 Hab.

Densidad poblacional promedio por edificacin

5,250

3.95

H22/ TOTAL VIVIENDAS

Dotacin para el AO 2009 en base al consumo por tipo de edificacin

133.91

(x C.R:Includo 17% de prdidas y desperdicios en la red de reciente instalacin)

Dotacin para el AO 2009 en base al consumo por habitante

150.63

(xC.C:Incidencia futura de incremento de consumo por la tendencia de crecimiento comercial)

TOMANDO EL VALOR MAYOR DE DOTACIN TENDREMOS


Consumo promedio instantaneo para la zona A

9.15

(RED EXISTENTE)

ZONA B :

Densidad poblacional promedio :

DATOS

TIPO1( unid.)=

200

Lit./viv./da

Lit./hab./da

Lit./viv./da

Lit./hab./da

Lit./viv./da

Lit./hab./da

140
120
100

1,180
1,020
948

200
150
130

1,150
1,050
888

190
175
150

Mximo
1,200
Mnimo
1,000
Promedio
950
TOTAL VIVIENDAS

7.23 habitantes por vivienda


TIPO2( unid.) =

2,420

Consumo promedio por edificacin/ da =

Poblacin en el Ao 2009:

500

TIPO3( unid.) =

800

2,000 x1,850 1,500 x 1,648 800x 1,370 620x 1,141


( 2,0001,500 800 620 )

50.00%

35,000 Hab.

Densidad poblacional promedio por edificacin


Dotacin para el AO 2009 en base al consumo por tipo de edificacin

17,500

7.23

131.13

(x C.R:Includo 45% de prdidas y desperdicios en la red de instalacin antigua)

Dotacin para el AO 2009 en base al consumo por habitante

148.58

(xC.C:Incidencia futura de incremento de consumo al mantener la tendencia de crecimiento sostenido de tipo residencial)

TOMANDO EL VALOR MAYOR DE DOTACIN TENDREMOS


Consumo promedio instantaneo para la zona B

30.09

(RED EN PROYECCIN)

ZONA C :

Densidad poblacional promedio :

DATOS

TIPO1( unid.)=

800

Lit./viv./da

Lit./hab./da

Lit./viv./da

Lit./hab./da

Lit./viv./da

Lit./hab./da

500
400
350

120
100
80

500
450
400

190
150
135

500
480
466

150
125
112

Mximo
Mnimo
Promedio

4.02 habitantes por vivienda


TIPO2( unid.) =

750

FACULTAD DE INGENIERIA CIVIL Y ARQUITECTURA

TIPO3( unid.) =

620

ESCUELA ACDEMICA PROFESIONAL DE INGENIERA CIVIL

SOLUCIONARIO DEL EXAMEN DE SUBSANACIN(CICLO 2.2009 (14/03/10) )


RESP.DEL REA HIDRAULICA :
ASIGNATURA

TOTAL VIVIENDAS

3,050

Consumo promedio por edificacin/ da =

Poblacin en el Ao 2009:

Ing. Civil Clifton Paucar y Montenegro - REG. CIP. 45773


ABASTECIMIENTOS DE AGUA Y ALCANTARILLADO - CIC404

1,300 x 650 1,800 x 600 1,200 x 500 730 x 350


(1,300 1,800 1,200 730 )

35.00%

35,000 Hab.

Densidad poblacional promedio por edificacin


Dotacin para el AO 2009 en base al consumo por tipo de edificacin

12,250

4.02

106.75

(x C.R:Includo 17% de prdidas y desperdicios en la red de reciente instalacin)

Dotacin para el AO 2009 en base al consumo por habitante

101.47

(xC.C:Incidencia futura de incremento de consumo por la tendencia de crecimiento comercial)

TOMANDO EL VALOR MAYOR DE DOTACIN TENDREMOS


Consumo promedio instantaneo para la zona C

15.14

Rpta. A y B
ZONA CON SERVICIO DE AGUA

ZONA SIN SERVICIO DE AGUA

CUADRO DE RESPUESTAS A Y B.DEMANDA CATASTRAL PROMEDIO:

Zona "A"

Zona "B"

Zona "C"

Densidad prom.poblac.(hab./edif.)

3.95

7.23

4.02

Poblacin Total ao 2009 ( habitantes )


Dotacin para el ao 2009 (Lt./hab./da)

5,250
150.63

17,500
148.58

12,250
106.75

9.15

30.09

15.14

Consumo prom.ao 2009 ( Lt./seg. )


Q de diseo del sistema ao 1995(Dato)

551.00

Rpta. C
CUADRO DE RESPUESTA PARA EL INCISO C.Zona "A"

Zona "B"

Zona "C"

Densidad prom.poblac.(hab./edif.)

3.95

7.23

4.02

Poblacin Total ao 2009 ( habitantes )

5,250

17,500

12,250

Dotacin para el ao 2009 calculada por edificacin sin


prdidas(Lt./hab./da)
Dotacin para el ao 2009 Inc. Prdidas segn datos suministrados

150.63

148.58

106.75

271.13

260.02

148.38

271.13

260.02

148.38

271.13

260.02

148.38

271.13

260.02

148.38

9.15

30.09

15.14

16.48

52.67

21.04

5,416.00

20,377.00

13,111.00

9.44

35.04

16.20

17.00

61.32

22.52

DEMANDA CATASTRAL POR EDIFICACIN Includo/PRDIDAS:

80%-75%-39% (Lt./hab./da)
Dotacin para el ao 2010 Inc. Prdidas segn datos proyectados 80%-75%-39%
(Lt./hab./da)
Dotacin para el ao 2021 Inc. Prdidas segn datos proyectados 60%-45%-17%
(Lt./hab./da)
Dotacin para el ao 2031 Inc. Prdidas segn datos proyectados 60%-45%-17%
(Lt./hab./da)

Consumo prom.ao 2009 sin / Prdidas (Lt./seg. )


Consumo prom.ao 2009 includo/ Prdidas (Lt./seg. )
Poblacin proyectada ao 2010
Consumo prom.ao 2010sin / Prdidas (Lt./seg. )
Consumo prom.ao 2010 Includo / Prdidas (Lt./seg. )
Poblacin proyectada ao 2021

7,348.00

83,363.00

24,424.00

Consumo prom.ao 2021 sin / Prdidas (Lt./seg. )

12.81

143.36

30.18

Consumo prom.ao 2021 Includo / Prdidas (Lt./seg. )

23.06

250.88

41.95

Poblacin proyectada ao 2031

9,300.00

445,066.00

38,565.00

Consumo prom.ao 2031sin / Prdidas (Lt./seg. )

16.21

765.37

47.65

Consumo prom.ao 2031 Includo / Prdidas (Lt./seg. )

29.18

1,339.40

66.23

Q de diseo del sistema ao 1995(Dato)

551.00

EL ANLISIS DE LA VARIACIN DE CONSUMO EN EL TIEMPO SE EFECTUAR CON LA DECISIN DE USAR PARA EL CALCULO DE LA DOTACIN INCREMENTA
ECUACIN LINEAL CON LOS DATOS (ZONAS URBANAS-DONDE EL INCREMENTO DEL CONSUMO NO SOLO DEPENDE DEL CRECIMIENTO POBLACIONAL) O EL
MATEMATICO DE CAPEN (ZONAS RURALES-DONDE LA DOTACIN SE BASA EN EL USO DOMESTICO Y SU VARIACIN DEPENDE DEL INCREMENTO POBL
PARA LO CUL ANTES DEBEMOS ESTIMAR EL QPROMEDIO DE DISEO SEGN LAS CARACTERISTICAS DE CONSUMO DIARIO Y HORARIO, LOS FACT
MAYORACIN K1 Y K2 Y LA POBLACIN FUTURA

ii)

Efectuamos las estimaciones de las demanda por variaciones de consumo diarias en funcin a los datos suministrados del diario
promedio horario y del diario maximo horario en Lt./seg., a fin de determinar los valores de K2.

Para determinar el caudal medio en Lt./seg. durante el ao 2009, se proceder a desarrollar el cuadro que contiene las variaciones horarias del dia de maximo consumo
ao, que representa en forma analitica a la curva masa (Produccin y Demanda vrs. Tiempo), teniendo como base que se consume 54 Lt./seg. (2009), y que es la produccin
captacin (100%) que es de 551 Lt.seg. con el cul se diseo el reservorio en el ao 1995, proyecto que entr en funcionamiento el ao 2001 con un Pd=15 aos, debiendo cu
eficiencia la demanda hasta el ao 1996, PERO SE OBSERVA EN EL CUADRO 01 que slo cumple con la demanda del ao 2009 PARA LAS ZONA A Y B, Y
INCLUIRSE LA DEMANDA INSATISFECHA EN C QUE NO CUENTA CON SISTEMA, EXISTE UN DEFICIT EN ESTE AO 2009 DE 44.4% .

DATO:

Qdisp. Mx.=

con caudal diseo de la obra existente=


Q x 2horas x 60 minnutos x 60 segundos

1,551 Litros/seg.
25 % del consumo 2009 sin perdidas porque?

HORAS

LOCALIDAD ( Consumo promedio ao 2009 )

CUADRO 02.-

GASTO
PROM.
@ 2 horas
% del Qprom.

VOLUM.(2009)

PRODUCC.(2009)

VOLUM. DE

PROM @ 2HR.

VOLUMEN

ACUM.CONSUMO

ACUM.24Hr.

CONS.RESERV.

(54.38 Lt./seg.)

(54.38 Lt./seg.)

(54.38 Lt./seg.)

(551 Lt./seg.)

(Demanda 54.38)

(Lt./seg.)

(Lts./s.)

(Lts.)

(Lts.)

(Lts.)

(Lts.)

GASTO

col.G-col.F

TENER CUIDADO CON LA RESTA


SIEMPRE ES (MAYOR-MENOR)

CON A y B obras c/sist

30.00%

16.314

127,249

127,249

35.00%

19.033

156,614

283,864

7,934,400

7,650,536

45.00%

24.471

293,652

577,516

11,901,600

11,324,084

105.00%

57.099

528,574

1,106,090

15,868,800

14,762,710

165.00%

89.727

557,939

1,664,029

19,836,000

18,171,971

10

120.00%

65.256

626,458

2,290,487

23,803,200

21,512,713

12

200.00%

108.760

616,669

2,907,156

27,770,400

24,863,244

14

115.00%

62.537

411,113

3,318,269

31,737,600

28,419,331

16

95.00%

51.661

469,843

3,788,112

35,704,800

31,916,688

18

145.00%

78.851

597,092

4,385,204

39,672,000

35,286,796

20

160.00%

87.008

479,632

4,864,836

43,639,200

38,774,364

22

85.00%

46.223

225,133

5,089,969

47,606,400

42,516,431

24

30.00%

16.314

5,089,969

47,606,400

POR DA

706.94

PORQUE
(Lts./s.)

5,089,969

3,967,200

5,089.97

(Lts.)

(M3)

54.38

k2prom.09:

Reserv.551 l/s.

3,839,951

38,676,481

Vol.diseo inic.reservorio 95:

no suma la hora 24

no es el mayor

2.00

VALOR SIN APLICACIN DEL FACTOR DE MAYORACION K1 =1.3


38,676.5
Como se tiene como dato que la persistencia de consumo de los valores mximos diarios es de 10 a 13% se utilizar el cuadro No. 02 para estimar el valor de K2 y el Vo
consumo del reservorio principal de ampliacin. Se muestra los graficos de las curvas masa en ambos casos con fines de comparacin de los consumos.

FACULTAD DE INGENIERIA CIVIL Y ARQUITECTURA


ESCUELA ACDEMICA PROFESIONAL DE INGENIERA CIVIL

SOLUCIONARIO DEL EXAMEN DE SUBSANACIN(CICLO 2.2009 (14/03/10) )


RESP.DEL REA HIDRAULICA :
ASIGNATURA

50000000

Ing. Civil Clifton Paucar y Montenegro - REG. CIP. 45773


ABASTECIMIENTOS DE AGUA Y ALCANTARILLADO - CIC404

50000000

45000000
40000000
35000000
30000000

25000000
20000000
15000000
10000000

5000000
0
0

10

12

14

16

18

20

SIMULACIN PRODUCCion.(2009) (54.38 Lt./seg.) (Lts.)

PRODUCC.(2009) ACUM.24Hr. (551 Lt./seg.) (Lts.)


SIMULACIN PRODUCCion.(2009) (54.38 Lt./seg.) (Lts.)

PRODUCC.(2009) ACUM.24Hr. (186.3 Lt./seg.) (Lts.)


VOLUM. ACUMUL. Cons.da mx. (Lts.)

HORAS

LOCALIDAD (Consumo del da mximo ao 2009 )

GASTO
CUADRO 03.PROM.
GASTO
@ 2 horas
PROM @ 2HR.
% del Qprom.
(186.30 Lt./seg.)
(Lt./seg.)

VOLUM.

PRODUCC.(2009)

VOLUM. DE

VOLUMEN

ACUMUL.

ACUM.24Hr.

CONSUM.RES. (100%)

(186.30 Lt./seg.)

Cons.da mx.

(186.3 Lt./seg.)

Cons.da mx.

(Lts.)

(Lts.)

(Lts.)

(Lts.)

(Lts./s.)

TENER CUIDADO RESTA

25.00%

13.60

107,676

107,676

3,967,200

30.00%

16.31

137,016

244,692

7,934,400

7,689,708

40.00%

21.75

274,068

518,760

11,901,600

11,382,840

100.00%

54.38

509,004

1,027,764

15,868,800

14,841,036

160.00%

87.01

528,588

1,556,352

19,836,000

18,279,648

10

110.00%

59.82

528,588

2,084,940

23,803,200

21,718,260

12

160.00%

87.01

538,380

2,623,320

27,770,400

25,147,080

14

115.00%

62.54

401,328

3,024,648

31,737,600

28,712,952

16

90.00%

48.94

450,252

3,474,900

35,704,800

32,229,900

18

140.00%

76.13

567,720

4,042,620

39,672,000

35,629,380

20

150.00%

81.57

450,252

4,492,872

43,639,200

39,146,328

22

80.00%

43.50

205,560

4,698,432

47,606,400

42,907,968

24

25.00%

13.60

4,698,432

47,606,400

42,907,968

4,698.43

Reserv(186.3 l./s.)=

-27,763,488

POR DA

652.56

4,698,432

(Lts./s.)

(Lts.)

54.38

Rpta. D:

k1=

1.30

(Reglam.)

(M3)

3,859,524

Vol.diseo ampliac.reservor 2008 por cons.mx.horar.

k2dis.(2008):

1.60 (CONSUMO PROMEDIO)

k2dis.09:

2.00

FACULTAD DE INGENIERIA CIVIL Y ARQUITECTURA


ESCUELA ACDEMICA PROFESIONAL DE INGENIERA CIVIL

SOLUCIONARIO DEL EXAMEN DE SUBSANACIN(CICLO 2.2009 (14/03/10) )


RESP.DEL REA HIDRAULICA :

Ing. Civil Clifton Paucar y Montenegro - REG. CIP. 45773

ASIGNATURA

ABASTECIMIENTOS DE AGUA Y ALCANTARILLADO - CIC404

CUADRO DE PRODUCCIN Y DEFICIT EN EL SERVICIO DE AGUA EN EL AO BASE DE OBSERVACION


PRODUCCIN Y DEFICIT DEL SERVICIO EN EL AO 2009

Producc.
(Lt./seg.)

PRODUCCIN

DATO
=

551.00

Caudal que se deriva a la zona A

9.15

sumas

9.15
1.66%

Caudal que se deriva a la zona B

30.09

Caudal que se deriva a la zona C

15.14

sumas

45.23

DEMANDA CATASTRAL CALCULADA

54.38

SUPERAVIT O DEFICIT EN EL AO 2008

913.24%
TOTAL HABITANTES:
total

HASTA EL AO 2043 DEL MES DE AGOSTOVER CUADRO

Rpta. E:

iii)

Efectuamos las estimaciones del crecimiento poblacional en funcin a la ecuacin caracteristicas de crecimiento poblacional
suministrados para cada zona, procesados utilizando el mtodo de incrementos porcentuales.

Yi 15.77 e

0.00005

Xi
CURVA DE CRECIMIENTO POBLACIONAL ZONA B

ZONA A :
CUADRO DE PROYECCION DE POBLACIN ZONA A

1984
1989
1994
1999
2004
2009
2014
2019
2024
2029
2034
2039
2044
2049
2054
2059
2064
2069
2074

%
CREC.POB.

kg(2014-2009)
kg(2019-2014)

TASAS CREC.
POB.

20,000

18,000

16,000

4,490
5,250
6,079
6,970
7,913
8,896
9,908
10,937
11,972
13,004
14,026
15,031
16,015
16,975
17,908

16.909735
15.791779
14.656435
13.527024
12.426353
11.374221
10.384034
9.465555
8.623659
7.858765
7.168163
6.548261
5.993284
5.497203

Habitantes con servicio en el 2009


kg(2009-2004)

POBLACIN

152
166
178
189
197
202
206
207
206
204
201
197
192
187

POBLACIN

AO

14,000

12,000

10,000

8,000

6,000

4,000

5,250

0.03128
0.02932
0.02736

2,000

0
2004 2009 2014 2019 2024 2029 2034 2039

POB.ESTIM. CUANDO ENTRE EN OPERACIN EL PROYECTO:


Pob.A(2013)

AO

5,914 Hab.

POBLACIN

TASAS CREC. POB.

FACULTAD DE INGENIERIA CIVIL Y ARQUITECTURA


ESCUELA ACDEMICA PROFESIONAL DE INGENIERA CIVIL

SOLUCIONARIO DEL EXAMEN DE SUBSANACIN(CICLO 2.2009 (14/03/10) )

ZONA B :

RESP.DEL REA HIDRAULICA :

Ing. Civil Clifton Paucar y Montenegro - REG. CIP. 45773

ASIGNATURA

ABASTECIMIENTOS DE AGUA Y ALCANTARILLADO - CIC404

Yi 75.55 0.00038 X i

CURVA DE CRECIMIENTO POBLACIONAL ZONA A

CUADRO DE PROYECCION DE POBLACIN ZONA B


AO

%
CREC.POB.

POBLACIN

TASAS CREC.
POB.

50,000,000

45,000,000

45,000,000

79.25956
82.20000
87.66630
98.28806
120.63700
175.02868
349.14506
1,304.38868
17,333.22150
###########
###########
###########
###########
###########

Habitantes con servicio en el 2009

9,762
17,500
31,885
59,837
118,650
261,786
719,987
3,233,786
45,414,925
7,917,284,469
238,210,793,572,546
#####################
#####################
#####################
#####################

1,548
2,877
5,590
11,763
28,627
91,640
502,760
8,436,228
1,574,373,909
47,640,575,257,615
####################
####################
####################
####################

40,000,000

35,000,000

POBLACIN

2004
2009
2014
2019
2024
2029
2034
2039
2044
2049
2054
2059
2064
2069
2074

30,000,000

25,000,000

20,000,000

15,000,000

10,000,000

5,000,000

0
0
2004

2009

2014

29,008 Hab.

POBLACIN

0.00002

ZONA C : Yi 44.88 e
CUADRO DE PROYECCION DE POBLACIN ZONA C
%

POBLACIN

CREC.POB.

2004
2009
2014
2019
2024
2029
2034
2039
2044
2049
2054
2059
2064
2069
2074
2079
2084
2089
2094
kg(2009-1994)
kg(2014-2009)
kg(2019-2014)

CURVA DE CRECIMIENTO POBLACIONAL ZONA A

TASAS CREC.
POB.

140,000

8,871
12,250
16,553
21,888
28,229
35,433
43,262
51,435
59,687
67,806
75,647
83,125
90,201
96,866
103,130
109,014
114,543
119,744
124,644

37.583768
35.127780
32.231114
28.969173
25.518719
22.094578
18.892259
16.043315
13.602482
11.563716
9.885334
8.512130
7.388851
6.466741
5.705281
5.071884
4.540932
4.092321

Habitantes con servicio en el 2009

TASAS CREC. POB.

Xi

676
861
1,067
1,268
1,441
1,566
1,635
1,650
1,624
1,568
1,496
1,415
1,333
1,253
1,177
1,106
1,040
980

120,000

100,000

POBLACIN

AO

2024

AO

POB.ESTIM. CUANDO ENTRE EN OPERACIN EL PROYECTO :


Pob.B(2013)

2019

0.06455
0.06021
0.05587

80,000

60,000

40,000

20,000

POB.ESTIM. CUANDO ENTRE EN OPERACIN EL PROYECTO :


Pob.C(2013)

15,694 Hab.
0
2004 2009 2014 2019 2024 2029 2034 2039 2044

AO

POBLACIN

FACULTAD DE INGENIERIA CIVIL Y ARQUITECTURA


ESCUELA ACDEMICA PROFESIONAL DE INGENIERA CIVIL

SOLUCIONARIO DEL EXAMEN DE SUBSANACIN(CICLO 2.2009 (14/03/10) )


RESP.DEL REA HIDRAULICA :
ASIGNATURA

Ing. Civil Clifton Paucar y Montenegro - REG. CIP. 45773


ABASTECIMIENTOS DE AGUA Y ALCANTARILLADO - CIC404

TASAS CREC. POB.

RptaE:

hasta el ao 2043 DEL MES DE AGOSTO

CUADRO DE POBLACIN Y CONSUMOS PROMEDIOS PROYECTADOS CON DEMANDA CONSTANTE


(Considerando que la demanda es cte. durante el periodo de diseo)
DEMANDA CATASTRAL:

Zona "A"

Zona "B"

Zona "C"

Poblacin Total ao 2009 ( habitantes )


Dotacin para el ao 2009(Lt./hab./da)
Consumo promedio ao 2008 ( Lt./seg. )

5,250
150.63
9.15

17,500
148.58
30.09

12,250
106.75
15.14

Poblacin Total ao 2013 ( habitantes )


dtacion 2013
Consumo promedio ao 2013 ( Lt./seg. )

5,914

29,008

15,694

10.31

49.88

19.39

Poblacin Total ao 2010 ( habitantes )


Consumo promedio ao 2010 ( Lt./seg. )

5,416
9.44

20,377
35.04

13,111
16.20

Poblacin Total ao 2021 ( habitantes )


Consumo promedio ao 2021 ( Lt./seg. )

7,348
12.81

83,363
143.36

24,424
30.18

Poblacin Total ao 2031 ( habitantes )


Consumo promedio ao 2031 ( Lt./seg. )

9,300
16.21

445,066
765.37

38,565
47.65

Poblacin Total ao 2036 ( habitantes )


Consumo promedio ao 2036 ( Lt./seg. )

10,320
17.99

1,725,507
2,967.31

46,532
57.49

Poblacin Total ao 2041 ( habitantes )


Consumo promedio ao 2041 ( Lt./seg. )

11,351
19.79

20,106,242
34,576.22

54,735
67.63

Poblacin Total ao 2169 ( habitantes )


Consumo promedio ao 2144 ( Lt./seg. )

35,675
62.20

Teniendo en cuenta que la demanda vara con respecto al tiempo efectuamos las correcciones
Usando la Expresin de CAPEN (Pg.7 AA-Simn Arocha) adaptada al dato base de cada zona
poblacional:

G = k p 0.125

Donde :

G = Consumo por habitante Lit/hab./da


P = Poblacin en miles
K = 204.4

Adaptando los valores de k para cada zona poblacional tendremos:

Zona A:

122.43

Zona B:

103.89

Con stos valores elaboramos el cuadro No. 8 corregido

CUADRO DE POBLACIN Y CONSUMOS PROMEDIOS PROYECTADOS CON DEMANDA VARIABLE


(Considerando que la demanda es variable durante el periodo de diseo, usando la Expresin de CAPEN, corregida)
DEMANDA CATASTRAL:

Zona "A"

Zona "B"

Zona "C"

Poblacin Total ao 2009 ( habitantes )


Consumo promedio ao 2009 ( Lt./seg. )

5,250
9.15

17,500
30.09

12,250
15.14

Poblacin Total ao 2013( habitantes )


Consumo promedio ao 2013 ( Lt./seg. )

5,914
10.47

29,008
53.14

15,694
20.00

Poblacin Total ao 2015 ( habitantes )


Consumo promedio ao 2015( Lt./seg. )

5,416
9.48

20,377
35.71

13,111
16.34

Poblacin Total ao 2021 ( habitantes )


Consumo promedio ao 2021 ( Lt./seg. )

7,348
13.36

83,363
174.24

24,424
32.90

Poblacin Total ao 2031 ( habitantes )


Consumo promedio ao 2031 ( Lt./seg. )

9,300
17.41

445,066
1,146.93

38,565
55.00

Poblacin Total ao 2036 ( habitantes )


Consumo promedio ao 2036 ( Lt./seg. )

10,320
19.58

1,725,507
5,267.34

46,532
67.93

Poblacin Total ao 2041 ( habitantes )


Consumo promedio ao 2041 ( Lt./seg. )

11,351
21.79

20,106,242
83,427.12

54,735
81.55

Poblacin Total ao 2015 ( habitantes )


Consumo promedio ao 2015( Lt./seg. )

CUADRO DE POBLACIN Y CONSUMOS PROMEDIOS PROYECTADOS CON DEMANDA VARIABLE

(Considerando que la demanda es variable segn una variacin lineal por tipo de consumo suministrada como dato obtenidad de la evaluacin cat
el estudio socioeconomico de desarrollo A- 150%, B-50%, C-35% cada 10 aos )
DEMANDA CATASTRAL:

INICIO DE
OPERACIN

1ERA. DECADA
DESDE EL AO
BASE DE OBSERV.

Zona "A"

Zona "B"

Zona "C"

Poblacin Total ao 1999 ( habitantes )


Dotacin para el ao 1999(Lt./hab./da)
Consumo promedio ao 1999 ( Lt./seg. )

0
75.32
0.00

0
0.00
0.00

K = 204.4
0.00
#VALUE!

Poblacin Total ao 2009 ( habitantes )


Dotacin para el ao 2009(Lt./hab./da)
Consumo promedio ao 2009 ( Lt./seg. )

5,250
150.63
9.15

17,500
148.58
30.09

12,250
106.75
15.14

Poblacin Total ao 2013 ( habitantes )


Dotacin para el ao 2013(Lt./hab./da)
Consumo promedio ao 2013 ( Lt./seg. )

5,914
195.82
13.40

29,008
163.44
54.87

15,694
114.22
20.75

Poblacin Total ao 2010 ( habitantes )


Dotacin para el ao 2010(Lt./hab./da)
Consumo promedio ao 2010( Lt./seg. )

5,416
286.20
17.94

20,377
193.15
45.55

13,111
129.17
19.60

Poblacin Total ao 2019 ( habitantes )


Dotacin para el ao 2019(Lt./hab./da)
Consumo promedio ao 2019( Lt./seg. )

6,970
376.58
30.38

59,837
222.87
154.35

21,888
144.11
36.51

Poblacin Total ao 2021 ( habitantes )


Dotacin para el ao 2021(Lt./hab./da)
Consumo promedio ao 2021 ( Lt./seg. )

7,348
489.55
41.63

83,363
245.16
236.54

24,424
154.20
43.59

Poblacin Total ao 2029( habitantes )


Dotacin para el ao 2029 (Lt./hab./da)
Consumo promedio ao 2029( Lt./seg. )

8,896
941.44
96.93

261,786
334.31
1,012.92

35,433
194.55
79.79

9,300
1,223.87
131.74

445,066
367.74
1,894.29

38,565
208.17
92.92

Poblacin Total ao 2031 ( habitantes )


FINAL DE
OPERACIN 15 Dotacin para el ao 2031(Lt./hab./da)
AOS
Consumo promedio ao 2031 ( Lt./seg. )
Poblacin Total ao 2039( habitantes )
Dotacin para el ao 2039 (Lt./hab./da)
Consumo promedio ao 2039( Lt./seg. )

poblacion
dotacion

ao 2043
2043
consumo 2043

2044
2044
2044
Poblacin Total ao 2049( habitantes )
Dotacin para el ao 2049 (Lt./hab./da)
Consumo promedio ao 2049( Lt./seg. )

Rpta. F:

10,937
3,233,786
51,435
2353.59375
501.4575 262.645031
297.93
18768.59
156.36
11765
3765.75
57.94
512.78 RPTA E
AO-2043
11,972
4118.789063
570.72
13,004
5883.984375
885.59

Hasta el MES DE JUNIO DEL ao 2024 para las zonas A, B y C, con el uso de la Represa + Prdid
FACULTAD DE INGENIERIA CIVIL Y ARQUITECTURA
ESCUELA ACDEMICA PROFESIONAL DE INGENIERA CIVIL

EXAMEN PARCIAL(CICLO 1.2009 (18/06/09) )


SOLUCIONARIO DEL EXAMEN DE SUBSANACIN(CICLO 2.2009 (14/03/10) )
ASIGNATURA

ABASTECIMIENTOS DE AGUA Y ALCANTARILLADO - CIC404

Para determinar el periodo de diseo de las obras complementarias usaremos los resultados de las estimaciones de los aos
cul el actual sistema cubre las necesidades de agua de las zonas "A", "B" y "C" segn la demanda y las ecuaciones del crec
poblacional planteados

iv)

SE CALCULA EL PERODO DE DISEO EN BASE A LOS DATOS CON QUE SE CUENTA, QUE INDICA QUE SON OBRAS DE AMPLIACIN PARA LAS ZONAS A, B Y C, POR LO QUE
UN VALOR VARIABLE PARA CADA CADA CASO UTILIZANDO EL MAYOR

USANDO LOS DOS TERMINOS DE LA FORMULA DE LAURIA:

X1 * = X 1 + X 2 = [( 2.6 ( 1 - a) ^ 1.12) / (r) ] + [ ( 0.3 (1 - a) Xo ^ 0.85 ) / (r)^ 1/2 ]


r= 0.2

Dato:

CONSIDERANDO LA SIGUIENTE DESCRIPCIN SIMPLIFICADA:

N.C.D. = NUEVA CAJA DE DISTRIBUCIN

N..C. = CAPTACIN NUEVA

NUEVO R.Z."B" = NUEVA RESERVORIO ZONAL "B"

N.L.C. = NUEVA LNEA DE CONDUCCIN

NUEVO R.Z."C" = NUEVO RESERVORIO ZONAL "C"

N.P.T.= NUEVA PLANTA DE TRATAMIENTO

AMP.R "A"= AMPLIACION DE REDES DE DISTRIB. EN A

N.R.P.=NUEVO RESERVORIO PRINCIPAL

AMP.R "B"= AMPLIACION DE REDES DE DISTRIB. EN B


R.N."C" = REDES NUEVAS PARA "C"

No. ORDEN

ESTRUCT.

C. EJEC.

COMPLEM.

( S/.)

01

N.C.

02

N.L.C.

03

N.P.T.

04

N.R.P.

05

N.C.D.

06

N.R.Z."A

07

N.R.Z."B

08

N.R.Z."C"

09

AMP.R "A"

10

R.N."B"

11

R.N."C"

C.OP.yMANT.
( S/.)

47,710,114
50,890,788
39,758,428
47,710,114
6,361,348
968,932
46,369,758
4,017,943
1,701,709
3,281,867
3,655,548

F. ESCAL.

TASA INT.

4,771,011
5,233,497
15,108,203
6,361,348
1,590,337
129,191
6,182,634
535,726
303,877
1,033,180
652,776
41,901,781

252,426,548

0.100
0.103
0.380
0.133
0.250
0.133
0.133
0.133
0.179
0.315
0.179

0.200
0.200
0.200
0.200
0.200
0.200
0.200
0.200
0.200
0.200
0.200

5.0 %

LOS COSTOS DE LAS OBRAS COMPLEMENTARIAS LO ESTIMAMOS EN BASE AL

Tasa activa :

%COSTO

0.189
0.202
0.158
0.189
0.025
0.004
0.184
0.016
0.007
0.013
0.014
1.000

DE REAJUSTE ANUAL Y EL CRECIMIENTO POBLACIO

20 %

FACULTAD DE INGENIERIA CIVIL Y ARQUITECTURA


ESCUELA ACDEMICA PROFESIONAL DE INGENIERA CIVIL

EXAMEN PARCIAL(CICLO 1.2009 (18/06/09) )


SOLUCIONARIO DEL EXAMEN DE SUBSANACIN(CICLO 2.2009 (14/03/10) )
ABASTECIMIENTOS DE AGUA Y ALCANTARILLADO - CIC404

ASIGNATURA

INICIAMOS EL CALCULO ASUMIENDO UN PERIODO DE DISEO DE 18 AOS ( 2029-2013), QUE ES EL DETERMINADO POR LA DISPONIBILIDAD DE CAUDAL DE LA REPRE
ABASTECER A LAS ZONAS A,B Y C, DE ACUERDO A SU CRECIMIENTO POBLACIONAL CORREGIDO POR EL MODELO LINEAL DE LOS DATOS SUMINISTRADOS INCLUDO P
CORRIGIENDO LOS COSTOS DE OPERACIN CON EL FACTOR DE 1.2 (18/15), AL HABERSE SUMINISTRADO PARA 15 AOS COMO DATO.

AO

Q (m3/seg.)

Cap. Reserv(m3)

1.-RELACIN ENTRE CAPTACIN Y NUEVA CAPTACIN PARALELA (N.C.)


COST.EJEC.
2,008
0.151
13,059
COST.MANT..
2,008
0.151
13,059

POB.(2009)

COSTO(Dato)

35,000
35,000

POB.(2031)

3,000,000
300,000

492,931
492,931

2.-RELACIN ENTRE LINEA DE CONDUCCIN Y LA NUEVA LNEA DE CONDUCCIN PARALELO(N.L.C.)


2,008
0.151
0
35,000
3,200,000
COST.MANT..
2,008
0.151
0
35,000
400,000

492,931
492,931

3.-RELACIN ENTRE PLANTA DE TRATAMIENTO Y LA NUEVA PLANTA DE TRATAMIENTO PARALELO(N.P.T.)


2,008
0.151
0
35,000
2,500,000
COST.MANT..
2,008
0.151
0
35,000
950,000

492,931
492,931

4.-RELACIN ENTRE RESERVORIO PRINCIPAL Y NUEVO RESERVORIO PRINCIPAL PARALELO(N.R.P.)


2,008
0.151
0
35,000
3,000,000
COST.MANT..
2,008
0.151
0
35,000
400,000

492,931
492,931

5.-RELACIN ENTRE CAJA DE DISTRIBUCIN Y NUEVA CAJA DE DISTRIBUCIN EN PARALELO


2,008
0.151
0
35,000
COST.MANT..
2,008
0.151
0
35,000

400,000
100,000

492,931
492,931

6.-RELACIN ENTRE RESERVORIO PRINCIPAL Y NUEVO RESERVORIO ZONAL EN A (N.R.Z.A.)


2,008
0.151
1,959
5,250
COST.MANT..
2,008
0.151
1,959
5,250

797,143
106,286

9,300
9,300

COST.EJEC.

COST.EJEC.

COST.EJEC.

COST.EJEC.

COST.EJEC.

7.-RELACIN ENTRE RESERVORIO PRINCIPAL Y NUEVO RESERVORIO ZONAL EN B (N.R.Z.B.)


2,008
0.151
6,530
17,500
COST.MANT..
2,008
0.151
6,530
17,500

38,148,514
5,086,469

445,066
445,066

8.-RELACIN ENTRE RESERVORIO PRINCIPAL Y NUEVO RESERVORIO ZONAL EN C (N.R.Z.C.)


2,008
0.151
4,571
12,250
COST.MANT..
2,008
0.151
4,571
12,250

3,305,571
440,743

38,565
38,565

5,250
5,250

1,400,000
250,000

9,300
9,300

COST.EJEC.

COST.EJEC.

9.-REDES EN A (R.D.N.A.)
COST.EJEC.

2,008

COST.MANT..

2,008

0.151
0.151

..

0.151
0.151

..
..

17,500
17,500

2,700,000
850,000

445,066
445,066

..

10.-REDES EN B (R.D.N.B.)
COST.EJEC.

2,008

COST.MANT..

2,008

11.-REDES EN C (R.D.N.C.)
COST.EJEC.

2,008

0.151

..

12,250

1,800,000

38,565

COST.MANT..

2,008

0.151

..

12,250

280,000

38,565

ESTIMAREMOS EL VALOR DE Xo PARA CADA ZONA SEGN EL CRECIMIENTO TEORICO ANUAL DE LA POBLACIN, TENIENDO EN CUENTA QUE EL SISTEMA ACTUAL SATISFACE A
POBLACION A, B Y C AL 100% DE SU NECESIDAD HASTA EL AO 2000 Y EL Pd inicial propuesto es igual al del sistema existente (1990-2010) = 20 aos (2011-2030)

Demanda en Deficit ( a2013 - a2009) / Crecimiento cte.de la demanda anual de diseo =

Xo =

Se considera el ao 2013, como aquel en donde empezar a operar el proyecto(Estudio y Ejecucin = 2 aos: 20110 y 20121)

ZONA A:

9,300 hab.
5,914 hab.
5,250 hab.

Pob.ref.(2031)=

2013 Pob.ref.(2013)=
2009 Pob.ref.(2009)=
DEFICIT periodo:

aos

2013-2009=

% incid. =
% incid. =
% incid. =

DEFICIT Poblacin:

hab.

R (crec.anual) =

16,442-16,100

664

R (crec.anual) = 342 / 4aos =


INTERCEPTO
DE LA
DEMANDA CON
Xo A =
EL EJE DE
ABCISAS
(Demanda =0)

DEFICIT Consum.en el periodo(Lit./da):


CRECIMIENTOCTE.DEMANDA DE DISEO
Dotacin para el ao
2013 Lt./hab./da

195.82
Demanda(2013)
Lt.//da

ZONA B:

130,023.8

Dotacin cte. Asumida del ao 2012


para el periodo de periodo (20092012) en Lt./hab./da
DOTACION(2036)
Demanda cte. (2009-2012) Lt./da

156,279

2013 Pob.ref.(2013)=
1996 Pob.ref.(1996)=
aos

941.44

445,066 hab.
29,008 hab.
17,500 hab.

Pob.ref.(2031)=

DEFICIT periodo:

DEFICIT Poblacin:

% incid. =
% incid. =
% incid. =
hab.

R (crec.anual) =

2012-1995=
17
48,565-44,275 =
11,508
DEFICIT Consum.en el periodo(Lit./da):
CRECIMIENTOCTE.DEMANDA DE DISEO
Dotacin para el ao
2013Lt./hab./da

163.44
Demanda(2011)
Lt.//da

ZONA C:

Dotacin cte. Asumida el ao


2010para el periodo de periodo (20092012) en Lt./hab./da
DOTACION(2036)
Demanda cte. (2000-2011) Lt./da

1,880,844.50

aos

334.31
226,305

2013 Pob.ref.(2012)=
1996 Pob.ref.(1990)=
DEFICIT Poblacin:

hab.

R (crec.anual) =

114.22
Demanda(2011)
Lt.//da

Dotacin cte. Asumida el ao 2010


para el periodo de periodo (19902011) en Lt./hab./da
DOTACION(2020)
Demanda cte. (2000-2011) Lt./da

1,792,607.92

179,606

TOTALES Pob.2031
Pob.2012
Pob.2009
No. ORDEN

ESTRUCT.

194.55

492,931 hab.
50,616 hab.
35,000 hab.
Xo

676.9

12.60% = r ABC
31.01%
0.00%
hab.

Xo C =

Xo prom.

X2 (S D)

hab.

R (crec.anual) = 22,207 / 11 aos =

INTERCEPTO
DE LA
DEMANDA
CON EL EJE
DE ABCISAS
(Demanda =0)

166.0

145.39% = r ABC
57.31% = r ABC
50.00%

% incid. =
% incid. =
% incid. =

2012-2001=
17
22,207-0 =
15,694
DEFICIT Consum.en el periodo(Lit./da):
CRECIMIENTOCTE.DEMANDA DE DISEO
Dotacin para el ao
2012 Lt./hab./da

hab.

R (crec.anual) = 4,290 / 17 aos =


INTERCEPTO
DE LA
DEMANDA CON
Xo B =
EL EJE DE
ABCISAS
(Demanda =0)

38,565 hab.
15,694 hab.
0 hab.

Pob.ref.(2031=

DEFICIT periodo:

3.04% = r ABC
11.68% = r ABC
15.00% = r ABC

923.2

10

6.33

X''(Pond.)

COMPLEM.

N.R.P.

6.33
6.33
6.33
6.33

2.90
2.89
2.00
2.79

0.55
0.58
0.31
0.53

05

N.C.D.

6.33

2.42

0.06

06

N.R.Z."A"

1.00

0.58

0.00

01

N.C.

02

N.L.C.

03

N.P.T.

04

EL SISTEMA PARA LA ZONA SATISFACE POR ENC


PERIODOD DE DISEO, POR LO QUE SERVIRA
ABASTECER INCLUSIVE A LA ZONA B Y C A FIN DE A
COSTOS EN INFRAESTRUCTURA EXCKUSIVA PARA ES
ULTIMAS ZONAS QUE EN EL 2007 O TIENEN SISTEMA

COSTOS EN INFRAESTRUCTURA EXCKUSIVA PARA ES


ULTIMAS ZONAS QUE EN EL 2007 O TIENEN SISTEMA

07

N.R.Z."B"

8.00

3.41

0.63

08

N.R.Z."C"

09

AMPLIAC.RD."A"

10

RD.N."B"

10.00
1.00
8.00

4.12
0.55
2.69

0.07
0.00
0.03

11

RD.N."C"

10.00

3.90

0.06

28.24

USANDO COSTOS DE OPERACIN Y MANTENIMIENTO DE


X1=
10.62
X2=
2.82

X=

13.44

2.82

SE ASUME = 15 AOS
2012 + 15 AOS = 2027

CALCULADO CON 15 AOS DE COSTOS DE OPE


MANTENIMIENTO

FACULTAD DE INGENIERIA CIVIL Y ARQUITECTURA


ESCUELA ACDEMICA PROFESIONAL DE INGENIERA CIVIL

SOLUCIONARIO DEL EXAMEN DE SUBSANACIN(CICLO 2.2009 (14/03/10) )


RESP.DEL REA HIDRAULICA :
ASIGNATURA

Ing. Civil Clifton Paucar y Montenegro - REG. CIP. 45773


ABASTECIMIENTOS DE AGUA Y ALCANTARILLADO - CIC404

Los volumenes de consumo del Reservorio principal y de los reservorios zonales "A", "B" y "C" se calculan en funcin al cuadro 2

RPTA H.-

El volumen inicialmente construdo ha sido calculado con el Q disponible de 151 l/s. con los datos del cuadro 02 de consumo promedio al tener como dato de persi
de consumos horarios entre los mximos y promedios de 10 a 13% en la parte inicial, cuando se determinaron los coeficientes de mayoracin.

Vi (2000) =
Coeficiente de variacin de consumo
CONSIDERANDO CON LA CONSTRUCCIN DE LAS O.C LA PRODUCCIN AL 100%
entre el 2008 y el 2026

ZONA
A

8.1250

2.4750

161,784

1.9501

210,319

Coef.CPROM.

4.1830

Del Cuadro 2 sumamos los valores absolutos mximos multiplicado por Cprom.( en m3)

13,059

MENOS EL VOL. RES. EFIC. AL 2008 =

197,260

DIF. PARA DISEO R. PRINC.COMP.2026


Resultados teoricos:

Para el reservorio zonal "B" que ser abastecido por el Reservorio Principal

CONJUNTO DE RESERVORIOS ZONALES

al no tener datos del consumo horario se optara por la recomendacin del RNC

ZONA A

Para Ao (2026) =

Solo para diferencia de Q(2025-1990)=

ZONA B

Para Ao (2026) =

L/s

#REF!

Para Ao (2026) =

40,916.66

25% ( 277.59 x 24 x 60 x 60) =


L/s

#REF!
ZONA C

2,845.58

25% ( 230.75 x 24 x 60 x 60)/1000 =

2,007.07

25% ( 91.43 x 24 x 60 x 60) =


#REF!

L/s

45,769.32

El predimensionamiento de las tuberas a partir de la caja de distribucin se tomaran del cuadro No. 09 para el ao 2021 y el calcu

RPTA H.-

Se deber verificar dos valores para tomar de ellos el mayor:


i)
ii )

( k1 x Qprom ) + Qincendio
K2 x Qprom.
DEL REGLAMENTO SE OBTIENE EL VALOR DE k1=1.3 AL NO EXISTIR DATOS DE VARIACIONES DIARIAS DEL AO 2007

DEL CUADRO 03 SE OBTIENE EL VALOR DE K2 para el da mximo anual = 3.4


USAMOS K 2 = 3.4 AL NO INDICARNOS LAS MEDIDAS DE CONTINGENICA PARA USAR EL VALOR DE Q MAX. PROM / QPROM QUE EVITARIA SOBREDIMENSIONAR
TUBERA CONSIDERANDO QUE LOS VALORES MXIMOS DEL CUADRO 03 SE PRESENTAN AL AO CON MENOS DE 30% DE PROBABILIDAD DE OCURRENCIA
USANDO LOS DATOS DE CAUDALES MEDIOS DEL CUADRO 9 PARA EL AO 2040 SE TIENE:

RELACIN DIAMETRO - VELOCIDAD ECONMICA


(Ref. : NORMAS INOS-VENEZOLANAS)

ZONA "A"

DIMETRO

( 1.3 x 107.31 ) + 50 =
ii ) 3.4 x 107.31
=
i)

ZONA "B"

189.50 Lit./seg.
364.82 Lit./seg.

D = 18"
D = 24"

mm.

Pulg.

75

3"

100

4"

150

6"

200

8"

( 1.3 X 858.86 ) + 50 =
ii ) 3.4 x 858.86
=
i)

1,166.52 Lit/seg.
2, 920.12 Lit./seg.

D=
D=

ZONA "C"
i ) ( 1.3 X 118,051.37 ) + 50 =

153,516.78 Lit/seg.

ii )

401,374.66 Lit./seg.

3.4 x 118,051.37

PULG.

SE UTILIZAN LOS DIAMETRO MAYORES ( NORMA.FAB. ITINTEC)

MM.

SE UTILIZAN LOS DIAMETRO MAYORES ( NORMA.FAB. ISO)

D=
D=

250

10"

300

12"

350

14"

400

16"

450

18"

500

20"

600

24"

700

30"

HOJA N 01/09

O JOSE

ncin al consumo promedio cuantificado por tipo

(RED EXISTENTE)
TIPO4( unid.) =

180

Lit./viv./da

Lit./hab./da

1,250
780
600

206
199
188

528.95 Lit./edif./da

hab.

hab
Lt/hab./da
150.63
Lt/hab./da

Lt./seg.

(RED EXISTENTE)
TIPO4( unid.) =

920

Lit./viv./da

Lit./hab./da

1,240
1,090
1,000

210
195
168

948.10 Lit./edif./da

hab.

hab
Lt/hab./da
148.58
Lt/hab./da

Lt./seg.

(RED EN PROYECCIN)
TIPO4( unid.) =

880

Lit./viv./da

Lit./hab./da

580
550
500

110
90
85

HOJA N 02/09

429.15 Lit./edif./da

hab.

hab
Lt/hab./da
106.75
Lt/hab./da

Lt./seg.

Total

DEFICIT./SUPERAV.

35,000.00

54.38

9.9%

Total

35,000.00

54.38
90.19

165.9%

38,904.00
60.68
100.84

166.2%

115,135.00
186.35
315.89

169.5%

492,931.00
829.23
1,434.81

173.0%

PROM.:

168.6%

PARA EL CALCULO DE LA DOTACIN INCREMENTADA , UNA


DEPENDE DEL CRECIMIENTO POBLACIONAL) O EL MODELO
U VARIACIN DEPENDE DEL INCREMENTO POBLACIONAL),
S DE CONSUMO DIARIO Y HORARIO, LOS FACTORES DE

cin a los datos suministrados del diario


de K2.

ariaciones horarias del dia de maximo consumo durante el


onsume 54 Lt./seg. (2009), y que es la produccin total de la
ento el ao 2001 con un Pd=15 aos, debiendo cumplir con
nda del ao 2009 PARA LAS ZONA A Y B, Y QUE DE
O 2009 DE 44.4% .

con caudal demanda -requerido...=Q x


2horas x 60 minnutos x 60 segundos

diseo de la obra existente=


x 60 minnutos x 60 segundos

nsumo promedio ao 2009 )

Q diseo = 1.3 X

54.38

70.69

PRODUCC.PROY.x1.3

VOLUM. DE

Lit./seg.

SIMULACIN

VOLUM. DE

VOLUM.

PRODUCCion.(2009)

CONS.RESERV.

ACUMUL. X 1.3

(54.38 Lt./seg.)

(Demanda 186.30 Lt./s)

CONSUMO (2009)

(Lts.)

(Lts.)

(Lts.)

(Lts.)

CON A y B obras c/sist+


C obra s/sist

col.I-col.F

col.F x 1.3

col.G x 1.3

ACUM.24Hr.2009
(C /242.19 Lit/seg.)

Caudal de diseo de la tubera de entrada del Rese

CONS.RESERVORIO

Usando la tabla que seala velocidades mximas y gastos m


cada dimetro, propuesta por las normas Venezolana

Mayorado c/ K1=1,3
(Lts.)

0
391,536

264,287

165,424

5,157,360

783,072

499,208

369,023

10,314,720

9,945,697

1,174,608

597,092

750,770

15,472,080

14,721,310

1,566,144

460,054

1,437,916

20,629,440

19,191,524

1,957,680

293,651

2,163,237

25,786,800

23,623,563

2,349,216

58,729

2,977,633

30,944,160

27,966,527

2,740,752

-166,404

3,779,302

36,101,520

32,322,218

3,132,288

-185,981

4,313,749

41,258,880

36,945,131

3,523,824

-264,288

4,924,545

46,416,240

41,491,695

3,915,360

-469,844

5,700,765

51,573,600

45,872,835

4,306,896

-557,940

6,324,286

56,730,960

50,406,674

4,698,432

-391,537

6,616,959

61,888,320

55,271,361

6,616,959

61,888,320

4,698,432
Reserv(54.38 l/s.)=

1,155,032

Vol.diseo ampliac.reservor 2008

38,676 X1.3

20"

4,991,936

REQUERIMIENTO DE DISEO (200


54.38

ANTERIOR DISEO (2000)


RESTA MAYOR-MENOR

55,271,361

-35,685,364
-35,685.4
50,279.4 M3

1,155.0

Diferencia generada por la produccin, que se entiende fsicamente que si no se cambia el dimetro de la tubera de acceso al Reserv
construir un RESERVORIO de (4,917.2-1,442.0=3,475.20 m3), considerando que en el ao 2000 se uso los mismos % de variaciones de co
considera el cambio de tubera de acceso para transportar 186.30 Lt./s. solo se necesitara u Reservorio de 1, 442 m3 que deber contrasta
consumo del reservorio construdo ( Evaluacin de costos y ventajas tcnicas)

M3

1,155.0

uadro No. 02 para estimar el valor de K2 y el Volumen de


mparacin de los consumos.

HOJA N 03/09

-37,521.4 M3
DEFICIT DE SERVICIO EN EL AO 2009 AL PRODUCIR 151 Lit./ seg. PARA CUBRIR UNA DEMANDA DE 186.30 Lit./s. NETA DE
CONSUMO POR EDIFICACIN SIN CONSIDERAR PERDIDAS+CONS.PBLICO+OPERAC.YMANTEN.VARIAC. DE DOTACIN
CON EL TIEMPO.

20

22

umo del da mximo ao 2009 )


CONSUM.RES. (100%)

iac.reservor 2008 por cons.mx.horar.

-27,763.00

(CONSUMO PROMEDIO)

M3

VARIA NOTORIAMENTE AL INCREMENTARSE LOS % DE VARIACIONDE CONSUMO MANTENIEN


1,155.0
-27,763.00
<
EL VOLUMEN DEL RESERVORIO EN EL 2031 SERA:
1,155.0

HOJA N 04/09

Habitantes
con servicio

Habitantes
sin servicio

1.3

1,501.5 M3

5,250

5,250
100.00%

0.00%

17,500

12,250
29,750

5,250

29,750

15.00%

85.00%

A:

5,250
35,000

UADRO

2 Ptos.

teristicas de crecimiento poblacional


uales.

CRECIMIENTO POBLACIONAL ZONA B

2039 2044 2049 2054 2059 2064 2069 2074

AO

TASAS CREC. POB.

HOJA N 05/09

CRECIMIENTO POBLACIONAL ZONA A

2024

2029

2034

2039

2044

AO

TASAS CREC. POB.

CRECIMIENTO POBLACIONAL ZONA A

AO

TASAS CREC. POB.

HOJA N 06/09

CONSTANTE
Total
35,000

AO DE
BASE DE FORM.

54.38
50,616
79.59
38,904
60.68
115,135
186.34
492,931
829.23

Qdisponible sistema includo represa


551 + 100 0= 1551 l/s.

1,782,359
3,042.80

442.71
2,600.08
2,157.37
1,714.66
1,271.94

20,172,328
34,663.64

POR AO
PARA 2035
PARA 2034
para 2033
PARA2032

36.89
1530.19359
1567.086345
1,603.98
1,677.76
1,714.66
2,636.98

POR MES
JULIO
AGOSTO
SET
NOVIEMBRE
DICIEMBRE
Jan-70

G = Consumo por habitante Lit/hab./da


P = Poblacin en miles

Zona C:

78.05

VARIABLE

PEN, corregida)
Total
35,000
54.38
50,616
83.60

INICIO DE
OPERACIN

38,904
61.53

115,135
220.50
492,931
1,219.34
1,782,359
5,354.85
20,172,328
83,530.46

Qdisponible sistema includo represa


551 + 1000 = 1551 l/s.
413.55
2,873.55
2,459.99
2,046.44
1,632.89
1,219.34

POR AO
PARA 2035
PARA 2034
para 2033
PARA2032
para 2031

34.46 POR MES


1,460.58
1,495.04
1,529.50
1563.967394

JULIO
AGOSTO
SET
OCT

1 AO MENOS QUE EL CUADRO D

VARIABLE

rada como dato obtenidad de la evaluacin catastral y

x f%Prdidas

Total

168.6%

#VALUE!
#VALUE!

#VALUE!

35,000
54.38

91.71

50,616
89.02

150.13

38,904
83.09

140.13

88,695
221.24

373.11

115,135
321.76

542.63

303.09 POR AO
1,148.80 PARA 2023

25.26 POR MES


1,502.40
Feb-29

1,451.89 PARA 2024


1,754.98 PARA 2025

1,603.43

306,115
1,189.64

2,006.26

492,931
2,118.95

Jun-29 7 AOS MENOS QUE EL CUADRO DE USO CON


6AOS MENOS QUE EL CUADRO DE USO VAR

3,573.49

3,296,158

Qdisponible sistema includo represa, considerando variacin tipificada socio econmica de la demand
551 +1000= 1551 l/s.
Qdisponible sistema includo represa, considerando slo variacin tipificada socio econmica de la demanda en el tiem

19222.88
4.828333333

AGOSTO
551.4066667

551 +1000= 1551 l/s.


179.72 POR AO
1,220.36 PARA 2026
1,579.79 PARA 2028

14.98 POR MES


1,594.77
Jan-31
1,609.75
Feb-31 4 AOS MENOS QUE EL CUADRO DE USO CON
3 AOS MENOS QUE EL CUADRO DE USO VAR

AO FINAL DE EFICIENCIA DEL CAUDAL DISPONIBLE VARIABLE EN


FUNCION DE: LA PRECISION EN LA PROYECCION DE LA DEMANDA
FUTURA Y LAS POLITICAS DE OPERACIN DEL SISTEMA QUE
DISMINUYAN LAS PERDIDAS DE LA RED.

n el uso de la Represa + Prdidas + Variac. de consumos

/09) )

HOJA N 07/09

ultados de las estimaciones de los aos hasta el


n la demanda y las ecuaciones del crecimiento

PLIACIN PARA LAS ZONAS A, B Y C, POR LO QUE Xo TIENE


.....Frmula General

VA CAJA DE DISTRIBUCIN

B" = NUEVA RESERVORIO ZONAL "B"

C" = NUEVO RESERVORIO ZONAL "C"

MPLIACION DE REDES DE DISTRIB. EN A

MPLIACION DE REDES DE DISTRIB. EN B


X1 (S D)

11.553
11.510
7.611
11.080
9.419
11.080
11.080
11.080
10.423
8.510
10.423
113.768

DATOS INICIALES DE COSTOS DE CONSTRUCCION Y OPERACIN (2008)


X1(Pond.)

2.18
2.32
1.20
2.09
0.24
0.04
2.04
0.18
0.07
0.11
0.15
10.62

No. ORDEN

ESTRUCT.

C. EJEC.

COMPLEM.

( S/.)

01

CAPTACIN

02

LNEA DE CONDUCCIN

03

PLANTA DE TRATAMIENTO

04

RESERVORIO PRINCIPAL

05

CAJA DE DISTRIBUCIN

06

REDES PARA "A"

07

REDES PARA "B"

08

REDES PARA "C"

C.OP.yMANT.
( S/.)

3,000,000
3,200,000
2,500,000
3,000,000
400,000
1,400,000
2,700,000
1,800,000

300,000
400,000
950,000
400,000
100,000
250,000
850,000
280,000

18,000,000

3,530,000

09
10
11

19.61%
DE REAJUSTE ANUAL Y EL CRECIMIENTO POBLACIONAL

/09) )

HOJA N 08/09

AO PARA EL QUE FUE EFICIENTE EL SISTEMA EXISTENTE QUE

R LA DISPONIBILIDAD DE CAUDAL DE LA REPRESA PARA


AL DE LOS DATOS SUMINISTRADOS INCLUDO PRDIDAS,
MO DATO.
CUBRE LA DEMANDA EL SAP QUE ENTRO EN OPERACIN EL AO 199O
Y CUYO Pd=20 aos, hasta el 2010.
COST.EJ.(Est.)

47,710,114
4,771,011

50,890,788
5,233,497

COST.MAN(Est.)

No en 1990 ya tenia un deficit de0% solo con A; en el 2008,se tiene un deficitit de 0% incluyendo A+B+C y de 0%
considerando solo A, se supone que no se penso adherir a la zona C y B al sistema y el 2009, se tendr un deficit
de 0% considerando A+B+C y 0% solo A.

FIN DEL PERIODO DE DISEO SUPUESTO DE FORMA PRELIMINAR

COSTO PROYECTADO AL 2013 (Ao en que entra en funcionamiento) (PPTO FINAL


REAJUSTADO POR LO QUE SE A USADO UN PERIODO DE AJUSTE DE t-to de 4
AOS , PARA APLICAR LA FORM. DE INTERES COMPUESTO (2013-2009)

Interes simple:

39,758,428
15,108,203

47,710,114
6,361,348

6,361,348
1,590,337

968,932
129,191

Interes compuesto:

T-TO =

2013

C = Co ( 1 + r (t-to) )
C = Co . r (t-to)
2009 =
4 aos
COSTOS DE CONSTRUCCION

46,369,758
6,182,634

4,017,943
535,726

1,701,709
303,877

COSTOS PROPORCIONALES A LAS OBRAS EXISTENTES DISEADAS EL 1995, EN FUNCION A LA


POBLACION SERVIDA CUYOS VALORES SE DISTORSIONAN EN BASE A LOS OBTENIDOS DEL
PLANEAMIENTO INICIAL

3,281,867
1,033,180

COSTOS REALES DE EJECUCIN Y MANTENIMIENTOS DE LAS REDES EN A SEGN


PLANEAMIENTO DE DISEO
Ref.:Red A

Ref.:Red B

2,187,911

3,655,548

7,049,986

340,342

652,776

2,219,440

N CUENTA QUE EL SISTEMA ACTUAL SATISFACE A LA


0-2010) = 20 aos (2011-2030)

de la demanda anual de diseo =


= r AB

= r AB

= r AB

unid. de tiempo

A PARA LA ZONA SATISFACE POR ENCIMA DEL


DE DISEO, POR LO QUE SERVIRA PARA
R INCLUSIVE A LA ZONA B Y C A FIN DE AHORRAR
N INFRAESTRUCTURA EXCKUSIVA PARA ESTAS DOS
ONAS QUE EN EL 2007 O TIENEN SISTEMA

N INFRAESTRUCTURA EXCKUSIVA PARA ESTAS DOS


ONAS QUE EN EL 2007 O TIENEN SISTEMA

OSTOS DE OPERACIN Y MANTENIMIENTO DE 20 AOS


PARA CUBRIR DEMANDA FUTURA (2012-2029)
PARA CUBRIR EL DEFICIT (2012-2009)

REDONDEO = 15
COMO LA REPRESA ABASTECE AL 2029 NO SERA NECESARIO UBICAR OTRA FUENTE PARA DIMENSIONAR HASTA EL 2027.
DEBERA PROGRAMARSE LA EJECUCIN POR ETAPAS EL PROYECTO DE AMPLIACION EN FUNCION A UN RECALCULO
POBLACIONAL EN BASE A UN ESTUDIO DE RENTABILIDAD( Recomendacin: Encontrar el punto de inflexin de densidad y
determinar la velocidad de crecimiento para determinar la poblacin mxima permisible por cada zona de ampliacin)

CALCULADO CON 15 AOS DE COSTOS DE OPERACIN Y


MANTENIMIENTO

HOJA N 07/07

" y "C" se calculan en funcin al cuadro 2

onsumo promedio al tener como dato de persistencia


eficientes de mayoracin.

13,059.0 M3
LA PRODUCCIN AL 100%

m3 - VOLUMEN DE CONSUMO NETO SIN MAYORACION (2025)


m3 - VOLUMEN DE CONSUMO NETO CON MAYORACION K1 = 1.3

EVALUAR SU APLICACIN COMPARANDO CON EL METODO

m3-VOLUMEN CONSIDERADO COMO DISEO DE CONSUMO EN 1995 CONSIDERANDO FACTOR DE MAYORACION K1=1.3(2,523X1.3)
m3 - VOLUMEN DE CONSUMO NETO CON MAYORACION K1 = 1.3

1,500.26 EVALUAR SU APLICACIN COMPARANDO CON EL METODO


USAR ESTE VALOR O.K.

el Reservorio Principal

a recomendacin del RNC


m3 - RESERV. Zonal "A"
ESTOS VALORES LOS COMPARAMOS CON LAS CURVAS MASA

m3 - RESERV. Zonal "B"

m3 - RESERV. Zonal "C"


M3

DESARROLLADAS PARA CADA ZONA CON LOS CONSUMOS


HORARIOS AFECTADOS POR LOS COEFICIENTES SUMINISTRADOS
COMO DATO, Fa= 1.45, Fb=0.85, Fc=0.70, A FIN DE OPTIMIZAR EL
DISEO PARA EL PERIODO DE 15 AOS OPTIMIZANDO EL
SISTEMA CON AJUSTES CATASTRALES DE CRECIMIENTO
MAXIMO Y DEFINICION DE ETAPAS DE DISEO

Rpta. H

cuadro No. 09 para el ao 2021 y el calculo de

ROM / QPROM QUE EVITARIA SOBREDIMENSIONAR LA


OS DE 30% DE PROBABILIDAD DE OCURRENCIA

CIN DIAMETRO - VELOCIDAD ECONMICA


(Ref. : NORMAS INOS-VENEZOLANAS)
V mx. (m/seg.)

Q mx. (LIT!SEG.

0.70

3.05

0.75

5.89

0.80

14.14

0.90

28.27

1.00

49.09

1.10

77.75

1.20

115.45

1.25

157.10

1.30

206.76

1.40

274.90

1.60

452.39

1.60

729.60

Rpta. H

CONSUMO DETERMINADO EN EL EXPEDIENTE DEL PROYECTO PARA DETERMINAR EL VOLUMEN DEL RESERVORIO EXISTENTE (

GASTO

de diseo de la tubera de entrada del Reservorio


la tabla que seala velocidades mximas y gastos mximos admisibles para
cada dimetro, propuesta por las normas Venezolanas (INOS-Ref.3)

CUADRO 03.-

LOCALIDAD (Consumo prom.usado en el diseo del proyecto(2000) con variaciones de consu


aos (2015))

PROM.(1999)
@ 2 horas
% del Qprom.

DE LA LOCAL.

(Lt./seg.)

(Lts./s.)

(Lts.)

menores

col.Vx Q c/sist(dato
151)

TENER CUIDADO CON


LA RESTA SIEMPRE ES
(MAYOR-MENOR)

0-2

10.00%

55.100

2-4

20.00%

4-6

30.00%

HORAS

GASTO
PROM @ 2HR.

VOLUM.
VOLUMEN

ACUM.24Hr.2015

CONSUMO (2015)

(C / 151Lit/seg.)

(Lts.)

REQUERIMIENTO DE DISEO (2009)


x 1.3 =
70.694 l/s
12"

ANTERIOR DISEO (2000)

551.00 l/s
30"

X 1.3 =

1,502 M3

RIR UNA DEMANDA DE 186.30 Lit./s. NETA DE


OPERAC.YMANTEN.VARIAC. DE DOTACIN

(Lts.)

595,080

595,080

4,198,620

110.200

991,800

1,586,880

8,397,240

165.300

2,380,320

3,967,200

12,595,860

6-8

90.00%

495.900

4,959,000

8,926,200

16,794,480

8-10

160.00%

881.600

5,752,440

14,678,640

20,993,100

10-12

130.00%

716.300

8,033,580

22,712,220

25,191,720

12-14

275.00%

1,515.250

8,628,660

31,340,880

29,390,340

14-16

160.00%

881.600

5,355,720

36,696,600

33,588,960

16-18

110.00%

606.100

5,157,360

41,853,960

37,787,580

18-20

150.00%

826.500

4,562,280

46,416,240

41,986,200

20-22

80.00%

440.800

2,677,860

49,094,100

46,184,820

22-24

55.00%

303.050

1,289,340

50,383,440

50,383,440

0-2

10.00%

55.100

POR DA

ambia el dimetro de la tubera de acceso al Reservorio se tendra que


o 2000 se uso los mismos % de variaciones de consumo del 2009; si se
ra u Reservorio de 1, 442 m3 que deber contrastarse con el volumen de
n de costos y ventajas tcnicas)

PRODUCC. PROY.

ACUMUL.

6,997.70
(Lts./s.)

50,383,440
(Lts.)

50,383,440

50,383,440

50,383.44

Reserv(343.68 l/s.)=

(M3)

Vol.diseo inic.reservorio 95:

583.14

Volumen de consumo del Reservorio si se utiliza los % de variacin del 2009


DIFERENCIA

NO ES NECESARIO INCREMENTAR EL RESERVORIO YA QUE EN EL 2009 SE SIGUE ABASTECIENDO


Qd=
45.23

DE DONDE SACA

E VARIACIONDE CONSUMO MANTENIENDOSE EL Q=186.3 Lit/seg. Cte. como produccin


Q demanda mxima horaria = 201.83

1 AO MENOS QUE EL CUADRO DE USO CONSTANTE DE LA DEMANDA

S MENOS QUE EL CUADRO DE USO CONSTANTE DE LA DEMANDA


MENOS QUE EL CUADRO DE USO VARIABLE DE LA DEMANDA USANDO EL MODELO DE CAPEN

n tipificada socio econmica de la demanda en el tiempo + p+erdidas en la red


socio econmica de la demanda en el tiempo

S MENOS QUE EL CUADRO DE USO CONSTANTE DE LA DEMANDA


S MENOS QUE EL CUADRO DE USO VARIABLE DE LA DEMANDA USANDO EL MODELO DE CAPEN

AS EL 1995, EN FUNCION A LA
ASE A LOS OBTENIDOS DEL

ACIN COMPARANDO CON EL METODO DIRECTO Y EL ANALISIS ECONOMICO DE LA OBRA

ACIN COMPARANDO CON EL METODO DIRECTO Y EL ANALISIS ECONOMICO DE LA OBRA

R EL VOLUMEN DEL RESERVORIO EXISTENTE ( 2000) CON PD=15 Aos

CONSUMO DETERMINADO EN BASE A LA PROYECCIN DE VARIACIONES DE CONSUMO Y EL CAUDAL


Q dis( 2031) =

131.74 litros/seg.

LOCALIDAD (Consumo prom.usado en el diseo del proyec

usado en el diseo del proyecto(2000) con variaciones de consumo del ao 1999 para las zonas A y B Pd=15
aos (2015)

GASTO
VOLUM. DE

CONSUMO REAL

CONS.RESERVORIO

ACUM.24Hr.2009

100% proy.demanda.2015 (C / 186.30 Lit/seg.)

(Lts.)

(Lts.)

col.Z-col.Y

CUADRO 03.-

100% demanda.2008

PROM.(2031)
@ 2 horas
% del Qprom.

DE LA LOCAL.

(Lts.)

(Lt./seg.)

(Lts./s.)

VOLUM. DE

HORAS

CONS.RESERVORIO

A y B Pd=15 ao

GASTO
PROM @ 2HR.

VOLUM.
VOLUMEN

ACUMUL.
CONSUMO (2008)

(Lts.)

(Lts.)

col.Z-col.AB

3,603,540

127,249

4,071,371

0-2

70.00%

92.218

663,970

663,970

6,810,360

283,864

8,113,376

2-4

70.00%

92.218

687,683

1,351,652

8,628,660

577,516

12,018,344

4-6

75.00%

98.805

758,822

2,110,475

7,868,280

1,106,090

15,688,390

6-8

85.00%

111.979

995,954

3,106,429

6,314,460

1,664,029

19,329,071

8-10

125.00%

164.675

1,185,660

4,292,089

2,479,500

2,290,487

22,901,233

10-12

125.00%

164.675

1,304,226

5,596,315

-1,950,540

2,907,156

26,483,184

12-14

150.00%

197.610

1,327,939

6,924,254

-3,107,640

3,318,269

30,270,691

14-16

130.00%

171.262

1,161,947

8,086,201

-4,066,380

3,788,112

33,999,468

16-18

115.00%

151.501

1,114,520

9,200,722

-4,430,040

4,385,204

37,600,996

18-20

120.00%

158.088

1,090,807

10,291,529

-2,909,280

4,864,836

41,319,984

20-22

110.00%

144.914

972,241

11,263,770

5,089,969

45,293,471

22-24

95.00%

125.153

782,536

12,046,306

5,089,969

45,293,471

0-2

70.00%

92.218

Reserv(343.68 l/s.)=

41,222,101

13,058,700
Vol.diseo inic.reservorio 95:

POR DA

Vol.diseo ampliac.reserv.2008

13,059.0

M3

1,155.0
-11,904.0

M3
M3

2009 SE SIGUE ABASTECIENDO

41,222.1

1,673.10
(Lts./s.)

12,046,306
12,046,306

12,046.31

(Lts.)

(M3)

139.42

Se calcula con ste dato (2031) por encontarse cercano al ao del periodo de diseo calcu

E VARIACIONES DE CONSUMO Y EL CAUDAL DE DISEO PARA EL AO 2031, USANDO PD=20 Aos (2012-2031)

RELACIN DIAMETRO - VELOCIDAD ECONMICA

LIDAD (Consumo prom.usado en el diseo del proyecto(2000) con variaciones de consumo del ao 1999 para las zonas
A y B Pd=15 aos (2015))
PRODUCC. PROY.
ACUM.24Hr.2008
(C / 151Lit/seg.)

VOLUM. DE

CONSUMO REAL

CONS.RESERVORIO

ACUM.24Hr.2009

(Lts.)

(Lts.)

V mx. (m/seg.)

mm.

Pulg.

100% demanda.2008

75

3"

0.70

(Lts.)

100

4"

0.75

CONS.RESERVORIO

100% proy.demanda.2015 (C / 186.30 Lit/seg.)

(Lts.)

(Ref. : NORMAS INOS-VENEZOLANAS)


DIMETRO

VOLUM. DE

150

6"

0.80

1,003,859

339,889

0
0

1,003,859

200

8"

0.90

2,007,718

656,065

2,007,718

250

10"

1.00

3,011,576

901,102

3,011,576

300

12"

1.10

4,015,435

909,006

4,015,435

350

14"

1.20

5,019,294

727,205

5,019,294

400

16"

1.25

6,023,153

426,838

6,023,153

450

18"

1.30

7,027,012

102,757

7,027,012

500

20"

1.40

8,030,870

-55,331

8,030,870

600

24"

1.60

9,034,729

-165,992

9,034,729

750

30"

1.60

10,038,588

-252,941

10,038,588

11,042,447

-221,323

11,042,447

12,046,306

12,046,306

12,046,306

Reserv(343.68 l/s.)=

12,046,306

12,046,306
1,154,042

Reserv(1003.28 l/s.)=

Vol.diseo inic.reservorio 95:

Vol.diseo ampliac.reserv.2008

1,154.04 M3

x 1.3 =

rcano al ao del periodo de diseo calculado de 15 aos (2013 + 15 =2028)

1,500.26 M3

ETRO - VELOCIDAD ECONMICA

ORMAS INOS-VENEZOLANAS)
Q (m3/seg.)
3.05
5.89

14.14
28.27
49.09
77.75
115.45
157.10
206.76
274.90
452.39
729.60

FACULTAD DE INGENIERIA CIVIL Y ARQUITECTURA


ESCUELA ACDEMICA PROFESIONAL DE INGENIERA CIVIL

SOLUCIONARIO DEL EXAMEN DE SUBSANACIN(CICLO 2.2009 (14/03/10) )


RESP.DEL REA HIDRAULICA :
ASIGNATURA

Ing. Civil Clifton Paucar y Montenegro - REG. CIP. 45773


ABASTECIMIENTOS DE AGUA Y ALCANTARILLADO - CIC404

HOJA N 01/09

RPTA A.-

i)

Iniciamos el proceso de estimacin calculando la dotacin promedio CATASTRAL, en funcin al consumo


promedio cuantificado por tipo de EDIFICACIN Y HABITANTE, as como el consumo promedio en Lt./seg.
80,500

Poblacin total localidad ALFA

Habitantes

ZONA A :

Densidad poblacional promedio :

DATOS

TIPO1( unid.)=

1,000

TIPO2( unid.) =

885

TIPO3( unid.) =

500

TIPO4( unid.) =

300

Lit./viv./da

Lit./hab./da

Lit./viv./da

Lit./hab./da

Lit./viv./da

Lit./hab./da

Lit./viv./da

Lit./hab./da

430
345
380

2,230
1,790
2,020

370
295
325

1,400
1,200
1,302

230
188
206

1,250
940
1,120

206
155
195

Mximo
2,600
Mnimo
2,100
Promedio
2,300
TOTAL VIVIENDAS

2,685

Consumo promedio por edificacin/ da =

Poblacin en el Ao 2009:

(RED EXISTENTE)

6.00 habitantes por vivienda

1,000 x 2,300 885 x 2,020 500 x 1,250 300 x 1,118


(1,000865 500300 )

20.00%

80,500 Hab.

Densidad poblacional promedio por edificacin


Dotacin para el AO 2009 en base al consumo por tipo de edificacin

1,890.02 Lit./edif./da

16,100 hab.

6.00 hab

315.00 Lt/hab./da

(x C.R:Includo 17% de prdidas y desperdicios en la red de reciente instalacin)

Dotacin para el AO 2009 en base al consumo por habitante

308.80 Lt/hab./da

(xC.C:Incidencia futura de incremento de consumo por la tendencia de crecimiento comercial)

TOMANDO EL VALOR MAYOR DE DOTACIN TENDREMOS


Consumo promedio instantaneo para la zona A

58.70 Lt./seg.

(RED EXISTENTE)

ZONA B :

Densidad poblacional promedio :

DATOS

TIPO1( unid.)=

2000

TIPO2( unid.) =

1500

TIPO3( unid.) =

800

TIPO4( unid.) =

620

Lit./viv./da

Lit./hab./da

Lit./viv./da

Lit./hab./da

Lit./viv./da

Lit./hab./da

Lit./viv./da

Lit./hab./da

240
190
200

1,810
1,450
1,648

200
160
180

1,750
1,220
1,370

190
135
150

1,400
1,040
1,141

150
115
128

Mximo
2,200
Mnimo
1,750
Promedio
1,850
TOTAL VIVIENDAS

4,920

Consumo promedio por edificacin/ da =


Poblacin en el Ao 2009:

9.00 habitantes por vivienda

2,000 x1,850 1,500 x 1,648 800x 1,370 620x 1,141


( 2,0001,500 800 620 )

55.00%

80,500 Hab.

Densidad poblacional promedio por edificacin


Dotacin para el AO 2009 en base al consumo por tipo de edificacin

1,621.02 Lit./edif./da

44,275 hab.

9.00 hab

180.11 Lt/hab./da

(x C.R:Includo 45% de prdidas y desperdicios en la red de instalacin antigua)

Dotacin para el AO 2009 en base al consumo por habitante

176.70 Lt/hab./da

(xC.C:Incidencia futura de incremento de consumo al mantener la tendencia de crecimiento sostenido de tipo residencial)

TOMANDO EL VALOR MAYOR DE DOTACIN TENDREMOS


Consumo promedio instantaneo para la zona B

92.30 Lt./seg.

(RED EN PROYECCIN)

ZONA C :

Densidad poblacional promedio :

DATOS

TIPO1( unid.)=

1300

TIPO2( unid.) =

1800

TIPO3( unid.) =

1200

TIPO4( unid.) =

730

Lit./viv./da

Lit./hab./da

Lit./viv./da

Lit./hab./da

Lit./viv./da

Lit./hab./da

Lit./viv./da

Lit./hab./da

700
600
650

200
165
180

700
550
600

190
150
165

700
450
500

180
125
132

550
350
350

140
90
100

Mximo
Mnimo
Promedio

4.00 habitantes por vivienda

FACULTAD DE INGENIERIA CIVIL Y ARQUITECTURA


ESCUELA ACDEMICA PROFESIONAL DE INGENIERA CIVIL

SOLUCIONARIO DEL EXAMEN DE SUBSANACIN(CICLO 2.2009 (14/03/10) )


RESP.DEL REA HIDRAULICA :

Ing. Civil Clifton Paucar y Montenegro - REG. CIP. 45773

ABASTECIMIENTOS DE AGUA Y ALCANTARILLADO - CIC404

ASIGNATURA

Consumo promedio por edificacin/ da =

Poblacin en el Ao 2009:

1,300 x 650 1,800 x 600 1,200 x 500 730 x 350


(1,300 1,800 1,200 730 )

25.00%

80,500 Hab.

Densidad poblacional promedio por edificacin


Dotacin para el AO 2009 en base al consumo por tipo de edificacin

HOJA N 02/09

552.78 Lit./edif./da

20,125 hab.

4.00 hab

138.20 Lt/hab./da

(x C.R:Includo 17% de prdidas y desperdicios en la red de reciente instalacin)

Dotacin para el AO 2009 en base al consumo por habitante

151.57 Lt/hab./da

(xC.C:Incidencia futura de incremento de consumo por la tendencia de crecimiento comercial)

TOMANDO EL VALOR MAYOR DE DOTACIN TENDREMOS


Consumo promedio instantaneo para la zona C

35.30 Lt./seg.

Rpta. A y B
ZONA CON SERVICIO DE AGUA

ZONA SIN SERVICIO DE AGUA

CUADRO DE RESPUESTAS A Y B.DEMANDA CATASTRAL PROMEDIO:

Zona "A"

Zona "B"

Zona "C"

Densidad prom.poblac.(hab./edif.)

6.00

9.00

4.00

Poblacin Total ao 2009 ( habitantes )


Dotacin para el ao 2009 (Lt./hab./da)

16,100
315.00

44,275
180.11

20,125
151.57

80,500.00

Consumo prom.ao 2009 ( Lt./seg. )

58.70

92.30

35.30

186.30

Q de diseo del sistema ao 2000(Dato)

Total

DEFICIT./SUPERAV.

123.4%

151.00

Rpta. C
CUADRO DE RESPUESTA PARA EL INCISO C.DEMANDA CATASTRAL POR EDIFICACIN
Includo/PRDIDAS:

Zona "A"

Zona "B"

Zona "C"

6.00

9.00

4.00

Poblacin Total ao 2009 ( habitantes )

16,100

44,275

20,125

Dotacin para el ao 2009 calculada por


edificacin sin prdidas(Lt./hab./da)

315.00

180.11

138.20

Dotacin para el ao 2009 Inc. Prdidas segn datos


suministrados 60%-45%-17% (Lt./hab./da)
Dotacin para el ao 2015 Inc. Prdidas segn datos
proyectados 60%-45%-17% (Lt./hab./da)
Dotacin para el ao 2021 Inc. Prdidas segn datos
proyectados 60%-45%-17% (Lt./hab./da)
Dotacin para el ao 2031 Inc. Prdidas segn datos
proyectados 60%-45%-17% (Lt./hab./da)

504.00

261.16

161.69

504.00

261.16

161.69

504.00

261.16

161.69

504.00

261.16

161.69

Consumo prom.ao 2009 sin / Prdidas


(Lt./seg. )
Consumo prom.ao 2009 includo/ Prdidas
(Lt./seg. )

58.70

92.30

32.19

183.19

93.92

133.83

37.66

265.41

Poblacin proyectada ao 2015

0.00

0.00

0.00

0.00

Consumo prom.ao 2015 sin / Prdidas


(Lt./seg. )
Consumo prom.ao 2015 Includo / Prdidas
(Lt./seg. )

0.00

0.00

0.00

0.00

0.00

0.00

0.00

0.00

Poblacin proyectada ao 2021

0.00

0.00

0.00

0.00

Consumo prom.ao 2021 sin / Prdidas


(Lt./seg. )
Consumo prom.ao 2021 Includo / Prdidas
(Lt./seg. )

0.00

0.00

0.00

0.00

0.00

0.00

0.00

0.00

Poblacin proyectada ao 2031

0.00

0.00

0.00

0.00

Consumo prom.ao 2031 sin / Prdidas


(Lt./seg. )
Consumo prom.ao 2031 Includo / Prdidas
(Lt./seg. )

0.00

0.00

0.00

0.00

0.00

0.00

0.00

0.00

#DIV/0!

PROM.:

#DIV/0!

Densidad prom.poblac.(hab./edif.)

Q de diseo del sistema ao 2000(Dato)

151.00

Total

80,500.00

144.9%

#DIV/0!

#DIV/0!

EL ANLISIS DE LA VARIACIN DE CONSUMO EN EL TIEMPO SE EFECTUAR CON LA DECISIN DE USAR PARA EL CALCULO DE LA
DOTACIN INCREMENTADA , UNA ECUACIN LINEAL CON LOS DATOS (ZONAS URBANAS-DONDE EL INCREMENTO DEL CONSUMO NO
SOLO DEPENDE DEL CRECIMIENTO POBLACIONAL) O EL MODELO MATEMATICO DE CAPEN (ZONAS RURALES-DONDE LA DOTACIN
SE BASA EN EL USO DOMESTICO Y SU VARIACIN DEPENDE DEL INCREMENTO POBLACIONAL), PARA LO CUL ANTES DEBEMOS
ESTIMAR EL QPROMEDIO DE DISEO SEGN LAS CARACTERISTICAS DE CONSUMO DIARIO Y HORARIO, LOS FACTORES DE
MAYORACIN K1 Y K2 Y LA POBLACIN FUTURA

ii)

Efectuamos las estimaciones de las demanda por variaciones de consumo diarias en funcin a los datos
suministrados del diario promedio horario y del diario maximo horario en Lt./seg., a fin de determinar los
valores de K2.

Para determinar el caudal medio en Lt./seg. durante el ao 2009, se proceder a desarrollar el cuadro que contiene las variaciones horarias del
consumo promedio durante el ao, que representa en forma analitica a la curva masa (Produccin y Demanda vrs. Tiempo), teniendo como base que
se consume 151.00 Lt./seg. (2009), y que es la produccin total de la captacin (100%) que es de 151 Lt.seg. con el cul se diseo el reservorio en el
ao 2000, proyecto que entr en funcionamiento el ao 2001 con un Pd=15 aos, debiendo cumplir con eficiencia la demanda hasta el ao 2015,
PERO SE OBSERVA EN EL CUADRO 01 que slo cumple con la demanda del ao 2009 PARA LAS ZONA A Y B, Y QUE DE INCLUIRSE LA
DEMANDA INSATISFECHA EN C QUE NO CUENTA CON SISTEMA, EXISTE UN DEFICIT EN ESTE AO 2009 DE 44.4% .

DATO:

HORAS

Qdisp. Mx.=

1,001 Litros/seg.

LOCALIDAD ( Consumo promedio ao 2009 )

GASTO

CUADRO 02.-

PROM.
@ 2 horas
% del Qprom.

GASTO

VOLUM.(2009)

PRODUCC.(2009)

VOLUM. DE

SIMULACIN

VOLUM. DE

PROM @ 2HR.

VOLUMEN

ACUM.CONSUMO

ACUM.24Hr.

CONS.RESERV.

PRODUCC.(2009)

CONS.RESERV.

(186.30 Lt./seg.)

(186.30 Lt./seg.)

(186.30 Lt./seg.)

(151 Lt./seg.)

(Demanda 186.3)

(186.30 Lt./seg.)

(Demanda 186.30 Lt./s)

(Lt./seg.)

(Lts./s.)

(Lts.)

(Lts.)

(Lts.)

(Lts.)

(Lts.)

(Lts.)

25.00%

46.575

368,874

368,874

1,087,200

718,326

1,341,360

972,486

30.00%

55.890

469,476

838,350

2,174,400

1,336,050

2,682,720

1,844,370

40.00%

74.520

938,952

1,777,302

3,261,600

1,484,298

4,024,080

2,246,778

100.00%

186.300

1,743,768

3,521,070

4,348,800

827,730

5,365,440

1,844,370

160.00%

298.080

1,810,836

5,331,906

5,436,000

104,094

6,706,800

1,374,894

10

110.00%

204.930

1,810,836

7,142,742

6,523,200

-619,542

8,048,160

905,418

12

160.00%

298.080

1,844,370

8,987,112

7,610,400

-1,376,712

9,389,520

402,408

14

115.00%

214.245

1,374,894

10,362,006

8,697,600

-1,664,406

10,730,880

368,874

16

90.00%

167.670

1,542,564

11,904,570

9,784,800

-2,119,770

12,072,240

167,670

18

140.00%

260.820

1,944,972

13,849,542

10,872,000

-2,977,542

13,413,600

-435,942

20

150.00%

279.450

1,542,564

15,392,106

11,959,200

-3,432,906

14,754,960

-637,146

22

80.00%

149.040

704,214

16,096,320

13,046,400

-3,049,920

16,096,320

24

25.00%

46.575

16,096,320

13,046,400

POR DA

2,235.60
(Lts./s.)

16,096,320

16,096.32

(Lts.)

(M3)

186.30

k2prom.09:

Reserv.151 l/s.

16,096,320
4,917,204

Vol.diseo inic.reservorio 95:

Reserv(186.3 l/s.)=

1,441,962

Vol.diseo ampliac.reservor 2008

1.60

VALOR SIN APLICACIN DEL FACTOR DE MAYORACION K1 =1.3


4,917.2
M3
1,442.0
Como se tiene como dato que la persistencia de consumo de los valores mximos diarios es de 10 a 13% se utilizar el cuadro No. 02 para estimar el
valor de K2 y el Volumen de consumo del reservorio principal de ampliacin. Se muestra los graficos de las curvas masa en ambos casos con fines
de comparacin de los consumos.

FACULTAD DE INGENIERIA CIVIL Y ARQUITECTURA


ESCUELA ACDEMICA PROFESIONAL DE INGENIERA CIVIL

SOLUCIONARIO DEL EXAMEN DE SUBSANACIN(CICLO 2.2009 (14/03/10) )


RESP.DEL REA HIDRAULICA :
ASIGNATURA

50000000
45000000
40000000

35000000
30000000
25000000

Ing. Civil Clifton Paucar y Montenegro - REG. CIP. 45773


ABASTECIMIENTOS DE AGUA Y ALCANTARILLADO - CIC404

HOJA N 03/09

25000000
20000000

15000000
10000000
5000000
0

10

12

14

16

18

20

22

SIMULACIN PRODUCCion.(2009) (54.38 Lt./seg.) (Lts.)


PRODUCC.(2009) ACUM.24Hr. (551 Lt./seg.) (Lts.)

SIMULACIN PRODUCCion.(2009) (54.38 Lt./seg.) (Lts.)


PRODUCC.(2009) ACUM.24Hr. (186.3 Lt./seg.) (Lts.)

VOLUM. ACUMUL. Cons.da mx. (Lts.)

HORAS

GASTO
CUADRO 03.PROM.
GASTO
@ 2 horas
PROM @ 2HR.
% del Qprom.
(186.30 Lt./seg.)
(Lt./seg.)

(Lts./s.)

LOCALIDAD (Consumo del da mximo ao 2009 )


VOLUM.

PRODUCC.(2009)

VOLUM. DE

VOLUMEN

ACUMUL.

ACUM.24Hr.

CONSUM.RES. (100%)

(186.30 Lt./seg.)

Cons.da mx.

(186.3 Lt./seg.)

Cons.da mx.

(Lts.)

(Lts.)

(Lts.)

(Lts.)

30.00%

55.89

435,960

435,960

1,087,200

651,240

35.00%

65.21

536,580

972,540

2,174,400

1,201,860
1,283,004

45.00%

83.84

1,006,056

1,978,596

3,261,600

105.00%

195.62

1,810,872

3,789,468

4,348,800

559,332

165.00%

307.40

1,911,456

5,700,924

5,436,000

-264,924

10

120.00%

223.56

2,146,176

7,847,100

6,523,200

-1,323,900

12

200.00%

372.60

2,112,660

9,959,760

7,610,400

-2,349,360

14

115.00%

214.25

1,408,464

11,368,224

8,697,600

-2,670,624

16

95.00%

176.99

1,609,668

12,977,892

9,784,800

-3,193,092

18

145.00%

270.14

2,045,592

15,023,484

10,872,000

-4,151,484

20

160.00%

298.08

1,643,184

16,666,668

11,959,200

-4,707,468

22

85.00%

158.36

771,300

17,437,968

13,046,400

-4,391,568

24

30.00%

55.89

17,437,968

13,046,400

-4,391,568

17,437.97

Reserv(186.3 l./s.)=

5,990,472

POR DA

2,421.94
(Lts./s.)

Rpta. D:

k1=

1.30

17,437,968
(Lts.)

(M3)

Vol.diseo ampliac.reservor 2008 por cons.mx.horar.

201.83

k2dis.(2008):

2.00 (Coef.da de mx.consumo)

(Reglam.)

k2dis.09:

1.60 (Consumo Promedio)

5,990.00

M3

1 Pto.
1 Pto.

1 Pto.

1 Pto.

SE USARA LA VARIACION LINEAL POR


QUE LA DIFERENCIA ES SIGNIFICATIVA
CON LA PROYECCION USANDO EL
MODELO MATEMATICO DE CAPEN

Q diseo = 1.3 X

186.30

242.19

VOLUM.

PRODUCC.PROY.x1.3

VOLUM. DE

Lit./seg.

ACUMUL. X 1.3
CONSUMO (2009)

ACUM.24Hr.2009
(C /242.19 Lit/seg.)

(Lts.)

(Lts.)

Caudal de diseo de la tubera de entrada del Reservorio

CONS.RESERVORIO

Usando la tabla que seala velocidades mximas y gastos mximos admisibles para
cada dimetro, propuesta por las normas Venezolanas (INOS-Ref.3)

Mayorado c/ K1=1,3

(Lts.)

=
479,536

1,413,360

933,824

1,089,855

2,826,720

1,736,865

2,310,493

4,240,080

1,929,587

4,577,391

5,653,440

1,076,049

6,931,478

7,066,800

135,322

9,285,565

8,480,160

-805,405

11,683,246

9,893,520

-1,789,726

13,470,608

11,306,880

-2,163,728

15,475,941

12,720,240

-2,755,701

18,004,405

14,133,600

-3,870,805

20,009,738

15,546,960

-4,462,778

20,925,216

16,960,320

-3,964,896

20,925,216

16,960,320

4,917 X1.3

20"
REQUERIMIENTO DE DISEO (2009)
186.30 x 1.3 =
242.19 l/s
20"

ANTERIOR DISEO (2000)

151.00 l/s
16"

-3,964,896

6,392,365
6,392.4 M3

6,392.4

1,442.0 X 1.3 =

1,875 M3

Diferencia generada por la produccin, que se entiende fsicamente que si no se cambia el dimetro de la tubera de acceso al Reservorio se tendra que
construir un RESERVORIO de (4,917.2-1,442.0=3,475.20 m3), considerando que en el ao 2000 se uso los mismos % de variaciones de consumo del 2009; si se
considera el cambio de tubera de acceso para transportar 186.30 Lt./s. solo se necesitara u Reservorio de 1, 442 m3 que deber contrastarse con el volumen de
consumo del reservorio construdo ( Evaluacin de costos y ventajas tcnicas)

-3,475.2 M3
DEFICIT DE SERVICIO EN EL AO 2009 AL PRODUCIR 151 Lit./ seg. PARA CUBRIR UNA DEMANDA DE 186.30 Lit./s. NETA DE
CONSUMO POR EDIFICACIN SIN CONSIDERAR PERDIDAS+CONS.PBLICO+OPERAC.YMANTEN.VARIAC. DE DOTACIN
CON EL TIEMPO.

DECIDIMOS INCREMENTAR E
POR LO QUE NECESITAMOS
NECESITAMOS CALCULAR LA

M3

2 Ptos.

VARIA NOTORIAMENTE AL INCREMENTARSE LOS % DE VARIACIONDE CONSUMO MANTENIENDOSE EL Q=186.3 Lit/seg. Cte. como
1,442.0
5,990.00
Q demanda mxima horaria = 20
<
EL VOLUMEN DEL RESERVORIO EN EL 2031 SERA:
1,442.0

1.3

1,874.6 M3

CONSUMO DETERMINADO EN EL EXPEDIENTE DEL PROYECTO PARA DETERMINAR EL VOLUMEN DEL RESERVORIO EXISTENTE ( 2000) CON PD=15 Aos

GASTO
HORAS

CUADRO 03.-

PROM.(1999)
@ 2 horas
% del Qprom.

DE LA LOCAL.

(Lt./seg.)

(Lts./s.)

LOCALIDAD (Consumo prom.usado en el diseo del proyecto(2000) con variaciones de consumo del ao 1999 para las zonas A y B
Pd=15 aos (2015))

GASTO
PROM @ 2HR.

VOLUM.
VOLUMEN

(Lts.)

PRODUCC. PROY.

VOLUM. DE

CONSUMO REAL

ACUMUL.

ACUM.24Hr.2015

CONS.RESERVORIO

ACUM.24Hr.2009

CONSUMO (2015)

(C / 151Lit/seg.)

100% proy.demanda.2015

(C / 186.30 Lit/seg.)

(Lts.)

(Lts.)

(Lts.)

(Lts.)

0-2

5.00%

7.550

108,720

108,720

1,087,200

978,480

2-4

15.00%

22.650

217,440

326,160

2,174,400

1,848,240

368,874
838,350

4-6

25.00%

37.750

597,960

924,120

3,261,600

2,337,480

1,777,302

6-8

85.00%

128.350

1,304,640

2,228,760

4,348,800

2,120,040

3,521,070

8-10

155.00%

234.050

1,522,080

3,750,840

5,436,000

1,685,160

5,331,906

10-12

125.00%

188.750

2,147,220

5,898,060

6,523,200

625,140

7,142,742

12-14

270.00%

407.700

2,283,120

8,181,180

7,610,400

-570,780

8,987,112

14-16

150.00%

226.500

1,386,180

9,567,360

8,697,600

-869,760

10,362,006

16-18

105.00%

158.550

1,359,000

10,926,360

9,784,800

-1,141,560

11,904,570

18-20

145.00%

218.950

1,168,740

12,095,100

10,872,000

-1,223,100

13,849,542

20-22

70.00%

105.700

652,320

12,747,420

11,959,200

-788,220

15,392,106

22-24

50.00%

75.500

298,980

13,046,400

13,046,400

16,096,320

0-2

5.00%

7.550

13,046,400

13,046,400

13,046.40

Reserv(343.68 l/s.)=

1,114,380

Reserv(343.68 l/s.)=

POR DA

1,812.00
(Lts./s.)

13,046,400
(Lts.)

(M3)

151.00

Volumen de consumo del Reservorio si se utiliza los % de variacin del 2009


DIFERENCIA

16,096,320

Vol.diseo inic.reservorio 95:

Vol.diseo ampliac.reserv.2008

1,114.0

M3

1,442.0
328.0

M3
M3

DECIDIMOS INCREMENTAR EN FORMA PARALELA UNA LINEA DE CONDUCCIN PARA INCREMENTAR EL CAUDAL DE 151 A 186.3, Qd= 35.3 Lt./seg.
POR LO QUE NECESITAMOS INCREMENTAR UN RESERVORIO DE 328 M3 PARA CUBRIR LA NECESIDAD SOLO GENERADA AL 2009
NECESITAMOS CALCULAR LA NECESIDAD PARA EL AO 2031 CONSIDERANDO UN PERIODO DE DISEO INICIAL DE 20 AOS

NDOSE EL Q=186.3 Lit/seg. Cte. como produccin


Q demanda mxima horaria = 201.83

ISTENTE ( 2000) CON PD=15 Aos

CONSUMO DETERMINADO EN BASE A LA PROYECCIN DE VARIACIONES DE CONSUMO Y EL CAUDAL DE DISEO PARA EL AO 2031, USANDO P
Q dis( 2026) =

0.00 litros/seg.

ones de consumo del ao 1999 para las zonas A y B

GASTO

Factor a =

100% demanda.2008

PROM.(2031)
@ 2 horas
% del Qprom.

DE LA LOCAL.

(Lts.)

(Lt./seg.)

(Lts./s.)

VOLUM. DE

HORAS

CONS.RESERVORIO

718,326
1,336,050
1,484,298
827,730
104,094
-619,542
-1,376,712
-1,664,406
-2,119,770
-2,977,542
-3,432,906
-3,049,920
-3,049,920

60.00%
65.00%
70.00%
85.00%
120.00%
115.00%
145.00%
125.00%
110.00%
115.00%
100.00%
90.00%
60.00%

4,917,204

4,917.2

GASTO

VOLUM.

PROM @ 2HR.

AUDAL DE 151 A 186.3, Qd= 35.3 Lt./seg.


O GENERADA AL 2009

(Lts.)

PRODUCC. PROY.

VOLUM. DE

ACUMUL.

ACUM.24Hr.2008

CONS.RESERVORIO

CONSUMO (2008)

(C / 151Lit/seg.)

100% proy.demanda.2015

(Lts.)

(Lts.)

(Lts.)
0

75.00%

0.000

2-4

81.25%

0.000

4-6

87.50%

0.000

6-8

106.25%

0.000

8-10

150.00%

0.000

10-12

143.75%

0.000

12-14

181.25%

0.000

14-16

156.25%

0.000

16-18

137.50%

0.000

18-20

143.75%

0.000

20-22

125.00%

0.000

22-24

112.50%

0.000

0-2

75.00%

0.000

0.00

Reserv(1003.28 l/s.)=

0.00
(Lts./s.)

VOLUMEN

0-2

POR DA

Vol.diseo ampliac.reserv.2008

1.25

LOCALIDAD (Consumo prom.usado en el diseo del proyecto(2000) con variaciones de consumo del ao 1999 para
las zonas A y B Pd=15 aos (2015) )
CUADRO 03.-

0
(Lts.)

0.00

(M3)

Vol.diseo inic.reservorio 2026:

0.00

Se calcula con ste dato (2031) por encontarse cercano al ao del periodo de diseo calculado de 15 aos (2012 + 15 =2027)

EL CAUDAL DE DISEO PARA EL AO 2031, USANDO PD=20 Aos (2012-2031)

RELACIN DIAMETRO - VELOCIDAD ECONMICA

diseo del proyecto(2000) con variaciones de consumo del ao 1999 para

s A y B Pd=15 aos (2015)

(Ref. : NORMAS INOS-VENEZOLANAS)


DIMETRO

M3

x 1.3 =

diseo calculado de 15 aos (2012 + 15 =2027)

0.00 M3

mm.

Pulg.

V mx. (m/seg.)

Q (m3/seg.)
3.05

75

3"

0.70

100

4"

0.75

5.89

150

6"

0.80

14.14

200

8"

0.90

28.27

250

10"

1.00

49.09

300

12"

1.10

77.75

350

14"

1.20

115.45

400

16"

1.25

157.10

450

18"

1.30

206.76

500

20"

1.40

274.90

600

24"

1.60

452.39

750

30"

1.60

729.60

FACULTAD DE INGENIERIA CIVIL Y ARQUITECTURA


ESCUELA ACDEMICA PROFESIONAL DE INGENIERA CIVIL

SOLUCIONARIO DEL EXAMEN DE SUBSANACIN(CICLO 2.2009 (14/03/10) )


RESP.DEL REA HIDRAULICA :
ASIGNATURA

Ing. Civil Clifton Paucar y Montenegro - REG. CIP. 45773


ABASTECIMIENTOS DE AGUA Y ALCANTARILLADO - CIC404

HOJA N 01/09

RPTA A.-

i)

Iniciamos el proceso de estimacin calculando la dotacin promedio CATASTRAL, en funcin al consumo


promedio cuantificado por tipo de EDIFICACIN Y HABITANTE, as como el consumo promedio en Lt./seg.
80,500

Poblacin total localidad ALFA

Habitantes

ZONA A :

Densidad poblacional promedio :

DATOS

TIPO1( unid.)=

1,000

TIPO2( unid.) =

885

TIPO3( unid.) =

500

TIPO4( unid.) =

300

Lit./viv./da

Lit./hab./da

Lit./viv./da

Lit./hab./da

Lit./viv./da

Lit./hab./da

Lit./viv./da

Lit./hab./da

430
345
380

2,230
1,790
2,020

370
295
325

1,400
1,200
1,302

230
188
206

1,250
940
1,120

206
155
195

Mximo
2,600
Mnimo
2,100
Promedio
2,300
TOTAL VIVIENDAS

2,685

Consumo promedio por edificacin/ da =

Poblacin en el Ao 2009:

(RED EXISTENTE)

6.00 habitantes por vivienda

1,000 x 2,300 885 x 2,020 500 x 1,250 300 x 1,118


(1,000865 500300 )

20.00%

80,500 Hab.

Densidad poblacional promedio por edificacin


Dotacin para el AO 2009 en base al consumo por tipo de edificacin

1,890.02 Lit./edif./da

16,100 hab.

6.00 hab

315.00 Lt/hab./da

(x C.R:Includo 17% de prdidas y desperdicios en la red de reciente instalacin)

Dotacin para el AO 2009 en base al consumo por habitante

308.80 Lt/hab./da

(xC.C:Incidencia futura de incremento de consumo por la tendencia de crecimiento comercial)

TOMANDO EL VALOR MAYOR DE DOTACIN TENDREMOS


Consumo promedio instantaneo para la zona A

58.70 Lt./seg.

(RED EXISTENTE)

ZONA B :

Densidad poblacional promedio :

DATOS

TIPO1( unid.)=

2000

TIPO2( unid.) =

1500

TIPO3( unid.) =

800

TIPO4( unid.) =

620

Lit./viv./da

Lit./hab./da

Lit./viv./da

Lit./hab./da

Lit./viv./da

Lit./hab./da

Lit./viv./da

Lit./hab./da

240
190
200

1,810
1,450
1,648

200
160
180

1,750
1,220
1,370

190
135
150

1,400
1,040
1,141

150
115
128

Mximo
2,200
Mnimo
1,750
Promedio
1,850
TOTAL VIVIENDAS

4,920

Consumo promedio por edificacin/ da =


Poblacin en el Ao 2009:

9.00 habitantes por vivienda

2,000 x1,850 1,500 x 1,648 800x 1,370 620x 1,141


( 2,0001,500 800 620 )

55.00%

80,500 Hab.

Densidad poblacional promedio por edificacin


Dotacin para el AO 2009 en base al consumo por tipo de edificacin

1,621.02 Lit./edif./da

44,275 hab.

9.00 hab

180.11 Lt/hab./da

(x C.R:Includo 45% de prdidas y desperdicios en la red de instalacin antigua)

Dotacin para el AO 2009 en base al consumo por habitante

176.70 Lt/hab./da

(xC.C:Incidencia futura de incremento de consumo al mantener la tendencia de crecimiento sostenido de tipo residencial)

TOMANDO EL VALOR MAYOR DE DOTACIN TENDREMOS


Consumo promedio instantaneo para la zona B

92.30 Lt./seg.

(RED EN PROYECCIN)

ZONA C :

Densidad poblacional promedio :

DATOS

TIPO1( unid.)=

1300

TIPO2( unid.) =

1800

TIPO3( unid.) =

1200

TIPO4( unid.) =

730

Lit./viv./da

Lit./hab./da

Lit./viv./da

Lit./hab./da

Lit./viv./da

Lit./hab./da

Lit./viv./da

Lit./hab./da

700
600
650

200
165
180

700
550
600

190
150
165

700
450
500

180
125
132

550
350
350

140
90
100

Mximo
Mnimo
Promedio

4.00 habitantes por vivienda

FACULTAD DE INGENIERIA CIVIL Y ARQUITECTURA


ESCUELA ACDEMICA PROFESIONAL DE INGENIERA CIVIL

SOLUCIONARIO DEL EXAMEN DE SUBSANACIN(CICLO 2.2009 (14/03/10) )


RESP.DEL REA HIDRAULICA :

Ing. Civil Clifton Paucar y Montenegro - REG. CIP. 45773

ABASTECIMIENTOS DE AGUA Y ALCANTARILLADO - CIC404

ASIGNATURA

Consumo promedio por edificacin/ da =

Poblacin en el Ao 2009:

1,300 x 650 1,800 x 600 1,200 x 500 730 x 350


(1,300 1,800 1,200 730 )

25.00%

80,500 Hab.

Densidad poblacional promedio por edificacin


Dotacin para el AO 2009 en base al consumo por tipo de edificacin

HOJA N 02/09

552.78 Lit./edif./da

20,125 hab.

4.00 hab

138.20 Lt/hab./da

(x C.R:Includo 17% de prdidas y desperdicios en la red de reciente instalacin)

Dotacin para el AO 2009 en base al consumo por habitante

151.57 Lt/hab./da

(xC.C:Incidencia futura de incremento de consumo por la tendencia de crecimiento comercial)

TOMANDO EL VALOR MAYOR DE DOTACIN TENDREMOS


Consumo promedio instantaneo para la zona C

35.30 Lt./seg.

Rpta. A y B
ZONA CON SERVICIO DE AGUA

ZONA SIN SERVICIO DE AGUA

CUADRO DE RESPUESTAS A Y B.DEMANDA CATASTRAL PROMEDIO:

Zona "A"

Zona "B"

Zona "C"

Densidad prom.poblac.(hab./edif.)

6.00

9.00

4.00

Poblacin Total ao 2009 ( habitantes )


Dotacin para el ao 2009 (Lt./hab./da)

16,100
315.00

44,275
180.11

20,125
151.57

80,500.00

Consumo prom.ao 2009 ( Lt./seg. )

58.70

92.30

35.30

186.30

Q de diseo del sistema ao 2000(Dato)

Total

DEFICIT./SUPERAV.

123.4%

151.00

Rpta. C
CUADRO DE RESPUESTA PARA EL INCISO C.DEMANDA CATASTRAL POR EDIFICACIN
Includo/PRDIDAS:

Zona "A"

Zona "B"

Zona "C"

6.00

9.00

4.00

Poblacin Total ao 2009 ( habitantes )

16,100

44,275

20,125

Dotacin para el ao 2009 calculada por


edificacin sin prdidas(Lt./hab./da)

315.00

180.11

138.20

Dotacin para el ao 2009 Inc. Prdidas segn datos


suministrados 60%-45%-17% (Lt./hab./da)
Dotacin para el ao 2015 Inc. Prdidas segn datos
proyectados 60%-45%-17% (Lt./hab./da)
Dotacin para el ao 2021 Inc. Prdidas segn datos
proyectados 60%-45%-17% (Lt./hab./da)
Dotacin para el ao 2031 Inc. Prdidas segn datos
proyectados 60%-45%-17% (Lt./hab./da)

504.00

261.16

161.69

504.00

261.16

161.69

504.00

261.16

161.69

504.00

261.16

161.69

Consumo prom.ao 2009 sin / Prdidas


(Lt./seg. )
Consumo prom.ao 2009 includo/ Prdidas
(Lt./seg. )

58.70

92.30

32.19

183.19

93.92

133.83

37.66

265.41

Poblacin proyectada ao 2015

0.00

0.00

0.00

0.00

Consumo prom.ao 2015 sin / Prdidas


(Lt./seg. )
Consumo prom.ao 2015 Includo / Prdidas
(Lt./seg. )

0.00

0.00

0.00

0.00

0.00

0.00

0.00

0.00

Poblacin proyectada ao 2021

0.00

0.00

0.00

0.00

Consumo prom.ao 2021 sin / Prdidas


(Lt./seg. )
Consumo prom.ao 2021 Includo / Prdidas
(Lt./seg. )

0.00

0.00

0.00

0.00

0.00

0.00

0.00

0.00

Poblacin proyectada ao 2031

0.00

0.00

0.00

0.00

Consumo prom.ao 2031 sin / Prdidas


(Lt./seg. )
Consumo prom.ao 2031 Includo / Prdidas
(Lt./seg. )

0.00

0.00

0.00

0.00

0.00

0.00

0.00

0.00

#DIV/0!

PROM.:

#DIV/0!

Densidad prom.poblac.(hab./edif.)

Q de diseo del sistema ao 2000(Dato)

151.00

Total

80,500.00

144.9%

#DIV/0!

#DIV/0!

EL ANLISIS DE LA VARIACIN DE CONSUMO EN EL TIEMPO SE EFECTUAR CON LA DECISIN DE USAR PARA EL CALCULO DE LA
DOTACIN INCREMENTADA , UNA ECUACIN LINEAL CON LOS DATOS (ZONAS URBANAS-DONDE EL INCREMENTO DEL CONSUMO NO
SOLO DEPENDE DEL CRECIMIENTO POBLACIONAL) O EL MODELO MATEMATICO DE CAPEN (ZONAS RURALES-DONDE LA DOTACIN
SE BASA EN EL USO DOMESTICO Y SU VARIACIN DEPENDE DEL INCREMENTO POBLACIONAL), PARA LO CUL ANTES DEBEMOS
ESTIMAR EL QPROMEDIO DE DISEO SEGN LAS CARACTERISTICAS DE CONSUMO DIARIO Y HORARIO, LOS FACTORES DE
MAYORACIN K1 Y K2 Y LA POBLACIN FUTURA

ii)

Efectuamos las estimaciones de las demanda por variaciones de consumo diarias en funcin a los datos
suministrados del diario promedio horario y del diario maximo horario en Lt./seg., a fin de determinar los
valores de K2.

Para determinar el caudal medio en Lt./seg. durante el ao 2009, se proceder a desarrollar el cuadro que contiene las variaciones horarias del
consumo promedio durante el ao, que representa en forma analitica a la curva masa (Produccin y Demanda vrs. Tiempo), teniendo como base que
se consume 151.00 Lt./seg. (2009), y que es la produccin total de la captacin (100%) que es de 151 Lt.seg. con el cul se diseo el reservorio en el
ao 2000, proyecto que entr en funcionamiento el ao 2001 con un Pd=15 aos, debiendo cumplir con eficiencia la demanda hasta el ao 2015,
PERO SE OBSERVA EN EL CUADRO 01 que slo cumple con la demanda del ao 2009 PARA LAS ZONA A Y B, Y QUE DE INCLUIRSE LA
DEMANDA INSATISFECHA EN C QUE NO CUENTA CON SISTEMA, EXISTE UN DEFICIT EN ESTE AO 2009 DE 44.4% .

DATO:

HORAS

Qdisp. Mx.=

1,001 Litros/seg.

LOCALIDAD ( Consumo promedio ao 2009 )

GASTO

CUADRO 02.-

PROM.
@ 2 horas
% del Qprom.

GASTO

VOLUM.(2009)

PRODUCC.(2009)

VOLUM. DE

SIMULACIN

VOLUM. DE

PROM @ 2HR.

VOLUMEN

ACUM.CONSUMO

ACUM.24Hr.

CONS.RESERV.

PRODUCC.(2009)

CONS.RESERV.

(186.30 Lt./seg.)

(186.30 Lt./seg.)

(186.30 Lt./seg.)

(151 Lt./seg.)

(Demanda 186.3)

(186.30 Lt./seg.)

(Demanda 186.30 Lt./s)

(Lt./seg.)

(Lts./s.)

(Lts.)

(Lts.)

(Lts.)

(Lts.)

(Lts.)

(Lts.)

25.00%

46.575

368,874

368,874

1,087,200

718,326

1,341,360

972,486

30.00%

55.890

469,476

838,350

2,174,400

1,336,050

2,682,720

1,844,370

40.00%

74.520

938,952

1,777,302

3,261,600

1,484,298

4,024,080

2,246,778

100.00%

186.300

1,743,768

3,521,070

4,348,800

827,730

5,365,440

1,844,370

160.00%

298.080

1,810,836

5,331,906

5,436,000

104,094

6,706,800

1,374,894

10

110.00%

204.930

1,810,836

7,142,742

6,523,200

-619,542

8,048,160

905,418

12

160.00%

298.080

1,844,370

8,987,112

7,610,400

-1,376,712

9,389,520

402,408

14

115.00%

214.245

1,374,894

10,362,006

8,697,600

-1,664,406

10,730,880

368,874

16

90.00%

167.670

1,542,564

11,904,570

9,784,800

-2,119,770

12,072,240

167,670

18

140.00%

260.820

1,944,972

13,849,542

10,872,000

-2,977,542

13,413,600

-435,942

20

150.00%

279.450

1,542,564

15,392,106

11,959,200

-3,432,906

14,754,960

-637,146

22

80.00%

149.040

704,214

16,096,320

13,046,400

-3,049,920

16,096,320

24

25.00%

46.575

16,096,320

13,046,400

POR DA

2,235.60
(Lts./s.)

16,096,320

16,096.32

(Lts.)

(M3)

186.30

k2prom.09:

Reserv.151 l/s.

16,096,320
4,917,204

Vol.diseo inic.reservorio 95:

Reserv(186.3 l/s.)=

1,441,962

Vol.diseo ampliac.reservor 2008

1.60

VALOR SIN APLICACIN DEL FACTOR DE MAYORACION K1 =1.3


4,917.2
M3
1,442.0
Como se tiene como dato que la persistencia de consumo de los valores mximos diarios es de 10 a 13% se utilizar el cuadro No. 02 para estimar el
valor de K2 y el Volumen de consumo del reservorio principal de ampliacin. Se muestra los graficos de las curvas masa en ambos casos con fines
de comparacin de los consumos.

FACULTAD DE INGENIERIA CIVIL Y ARQUITECTURA


ESCUELA ACDEMICA PROFESIONAL DE INGENIERA CIVIL

SOLUCIONARIO DEL EXAMEN DE SUBSANACIN(CICLO 2.2009 (14/03/10) )


RESP.DEL REA HIDRAULICA :
ASIGNATURA

50000000
45000000
40000000

35000000
30000000
25000000

Ing. Civil Clifton Paucar y Montenegro - REG. CIP. 45773


ABASTECIMIENTOS DE AGUA Y ALCANTARILLADO - CIC404

HOJA N 03/09

25000000
20000000

15000000
10000000
5000000
0

10

12

14

16

18

20

22

SIMULACIN PRODUCCion.(2009) (54.38 Lt./seg.) (Lts.)


PRODUCC.(2009) ACUM.24Hr. (551 Lt./seg.) (Lts.)

SIMULACIN PRODUCCion.(2009) (54.38 Lt./seg.) (Lts.)


PRODUCC.(2009) ACUM.24Hr. (186.3 Lt./seg.) (Lts.)

VOLUM. ACUMUL. Cons.da mx. (Lts.)

HORAS

GASTO
CUADRO 03.PROM.
GASTO
@ 2 horas
PROM @ 2HR.
% del Qprom.
(186.30 Lt./seg.)
(Lt./seg.)

(Lts./s.)

LOCALIDAD (Consumo del da mximo ao 2009 )


VOLUM.

PRODUCC.(2009)

VOLUM. DE

VOLUMEN

ACUMUL.

ACUM.24Hr.

CONSUM.RES. (100%)

(186.30 Lt./seg.)

Cons.da mx.

(186.3 Lt./seg.)

Cons.da mx.

(Lts.)

(Lts.)

(Lts.)

(Lts.)

30.00%

55.89

435,960

435,960

1,087,200

651,240

35.00%

65.21

536,580

972,540

2,174,400

1,201,860
1,283,004

45.00%

83.84

1,006,056

1,978,596

3,261,600

105.00%

195.62

1,810,872

3,789,468

4,348,800

559,332

165.00%

307.40

1,911,456

5,700,924

5,436,000

-264,924

10

120.00%

223.56

2,146,176

7,847,100

6,523,200

-1,323,900

12

200.00%

372.60

2,112,660

9,959,760

7,610,400

-2,349,360

14

115.00%

214.25

1,408,464

11,368,224

8,697,600

-2,670,624

16

95.00%

176.99

1,609,668

12,977,892

9,784,800

-3,193,092

18

145.00%

270.14

2,045,592

15,023,484

10,872,000

-4,151,484

20

160.00%

298.08

1,643,184

16,666,668

11,959,200

-4,707,468

22

85.00%

158.36

771,300

17,437,968

13,046,400

-4,391,568

24

30.00%

55.89

17,437,968

13,046,400

-4,391,568

17,437.97

Reserv(186.3 l./s.)=

5,990,472

POR DA

2,421.94
(Lts./s.)

Rpta. D:

k1=

1.30

17,437,968
(Lts.)

(M3)

Vol.diseo ampliac.reservor 2008 por cons.mx.horar.

201.83

k2dis.(2008):

2.00 (Coef.da de mx.consumo)

(Reglam.)

k2dis.09:

1.60 (Consumo Promedio)

5,990.00

M3

Yi 15.77 e

0.00005

Xi
CURVA DE CRECIMIENTO POBLACIONAL ZONA B

20,000

18,000

POBLACIN

16,000

14,000

12,000

10,000

8,000

6,000

4,000

2,000

0
2004 2009 2014 2019 2024 2029 2034 2039 2044 2049 2054 2059 2064 2069 2074

AO

POBLACIN

Yi 25.66 0.00015 X i

TASAS CREC. POB.

CURVA DE CRECIMIENTO POBLACIONAL ZONA A


1

POBLACIN

0
1

AO

Yi 77.17 e

0.00004

Xi

CURVA DE CRECIMIENTO POBLACIONAL ZONA A

CURVA DE CRECIMIENTO POBLACIONAL ZONA A


140,000

120,000

POBLACIN

100,000

80,000

60,000

40,000

20,000

0
200420092014201920242029203420392044

AO

POBLACIN

TASAS CREC. POB.

1 Pto.
1 Pto.

1 Pto.

1 Pto.

SE USARA LA VARIACION LINEAL POR


QUE LA DIFERENCIA ES SIGNIFICATIVA
CON LA PROYECCION USANDO EL
MODELO MATEMATICO DE CAPEN

Q diseo = 1.3 X

186.30

242.19

VOLUM.

PRODUCC.PROY.x1.3

VOLUM. DE

Lit./seg.

ACUMUL. X 1.3
CONSUMO (2009)

ACUM.24Hr.2009
(C /242.19 Lit/seg.)

(Lts.)

(Lts.)

Caudal de diseo de la tubera de entrada del Reservorio

CONS.RESERVORIO

Usando la tabla que seala velocidades mximas y gastos mximos admisibles para
cada dimetro, propuesta por las normas Venezolanas (INOS-Ref.3)

Mayorado c/ K1=1,3

(Lts.)

=
479,536

1,413,360

933,824

1,089,855

2,826,720

1,736,865

2,310,493

4,240,080

1,929,587

4,577,391

5,653,440

1,076,049

6,931,478

7,066,800

135,322

9,285,565

8,480,160

-805,405

11,683,246

9,893,520

-1,789,726

13,470,608

11,306,880

-2,163,728

15,475,941

12,720,240

-2,755,701

18,004,405

14,133,600

-3,870,805

20,009,738

15,546,960

-4,462,778

20,925,216

16,960,320

-3,964,896

20,925,216

16,960,320

4,917 X1.3

20"
REQUERIMIENTO DE DISEO (2009)
186.30 x 1.3 =
242.19 l/s
20"

ANTERIOR DISEO (2000)

151.00 l/s
16"

-3,964,896

6,392,365
6,392.4 M3

6,392.4

1,442.0 X 1.3 =

1,875 M3

Diferencia generada por la produccin, que se entiende fsicamente que si no se cambia el dimetro de la tubera de acceso al Reservorio se tendra que
construir un RESERVORIO de (4,917.2-1,442.0=3,475.20 m3), considerando que en el ao 2000 se uso los mismos % de variaciones de consumo del 2009; si se
considera el cambio de tubera de acceso para transportar 186.30 Lt./s. solo se necesitara u Reservorio de 1, 442 m3 que deber contrastarse con el volumen de
consumo del reservorio construdo ( Evaluacin de costos y ventajas tcnicas)

-3,475.2 M3
DEFICIT DE SERVICIO EN EL AO 2009 AL PRODUCIR 151 Lit./ seg. PARA CUBRIR UNA DEMANDA DE 186.30 Lit./s. NETA DE
CONSUMO POR EDIFICACIN SIN CONSIDERAR PERDIDAS+CONS.PBLICO+OPERAC.YMANTEN.VARIAC. DE DOTACIN
CON EL TIEMPO.

DECIDIMOS INCREMENTAR E
POR LO QUE NECESITAMOS
NECESITAMOS CALCULAR LA

M3

2 Ptos.

VARIA NOTORIAMENTE AL INCREMENTARSE LOS % DE VARIACIONDE CONSUMO MANTENIENDOSE EL Q=186.3 Lit/seg. Cte. como
1,442.0
5,990.00
Q demanda mxima horaria = 20
<
EL VOLUMEN DEL RESERVORIO EN EL 2031 SERA:
1,442.0

1.3

1,874.6 M3

CONSUMO DETERMINADO EN EL EXPEDIENTE DEL PROYECTO PARA DETERMINAR EL VOLUMEN DEL RESERVORIO EXISTENTE ( 2000) CON PD=15 Aos

GASTO
HORAS

CUADRO 03.-

PROM.(1999)
@ 2 horas
% del Qprom.

DE LA LOCAL.

(Lt./seg.)

(Lts./s.)

LOCALIDAD (Consumo prom.usado en el diseo del proyecto(2000) con variaciones de consumo del ao 1999 para las zonas A y B
Pd=15 aos (2015))

GASTO
PROM @ 2HR.

VOLUM.
VOLUMEN

(Lts.)

PRODUCC. PROY.

VOLUM. DE

CONSUMO REAL

ACUMUL.

ACUM.24Hr.2015

CONS.RESERVORIO

ACUM.24Hr.2009

CONSUMO (2015)

(C / 151Lit/seg.)

100% proy.demanda.2015

(C / 186.30 Lit/seg.)

(Lts.)

(Lts.)

(Lts.)

(Lts.)

0-2

5.00%

7.550

108,720

108,720

1,087,200

978,480

2-4

15.00%

22.650

217,440

326,160

2,174,400

1,848,240

368,874
838,350

4-6

25.00%

37.750

597,960

924,120

3,261,600

2,337,480

1,777,302

6-8

85.00%

128.350

1,304,640

2,228,760

4,348,800

2,120,040

3,521,070

8-10

155.00%

234.050

1,522,080

3,750,840

5,436,000

1,685,160

5,331,906

10-12

125.00%

188.750

2,147,220

5,898,060

6,523,200

625,140

7,142,742

12-14

270.00%

407.700

2,283,120

8,181,180

7,610,400

-570,780

8,987,112

14-16

150.00%

226.500

1,386,180

9,567,360

8,697,600

-869,760

10,362,006

16-18

105.00%

158.550

1,359,000

10,926,360

9,784,800

-1,141,560

11,904,570

18-20

145.00%

218.950

1,168,740

12,095,100

10,872,000

-1,223,100

13,849,542

20-22

70.00%

105.700

652,320

12,747,420

11,959,200

-788,220

15,392,106

22-24

50.00%

75.500

298,980

13,046,400

13,046,400

16,096,320

0-2

5.00%

7.550

13,046,400

13,046,400

13,046.40

Reserv(343.68 l/s.)=

1,114,380

Reserv(343.68 l/s.)=

POR DA

1,812.00
(Lts./s.)

13,046,400
(Lts.)

(M3)

151.00

Volumen de consumo del Reservorio si se utiliza los % de variacin del 2009


DIFERENCIA

16,096,320

Vol.diseo inic.reservorio 95:

Vol.diseo ampliac.reserv.2008

1,114.0

M3

1,442.0
328.0

M3
M3

DECIDIMOS INCREMENTAR EN FORMA PARALELA UNA LINEA DE CONDUCCIN PARA INCREMENTAR EL CAUDAL DE 151 A 186.3, Qd= 35.3 Lt./seg.
POR LO QUE NECESITAMOS INCREMENTAR UN RESERVORIO DE 328 M3 PARA CUBRIR LA NECESIDAD SOLO GENERADA AL 2009
NECESITAMOS CALCULAR LA NECESIDAD PARA EL AO 2031 CONSIDERANDO UN PERIODO DE DISEO INICIAL DE 20 AOS

NDOSE EL Q=186.3 Lit/seg. Cte. como produccin


Q demanda mxima horaria = 201.83

ISTENTE ( 2000) CON PD=15 Aos

CONSUMO DETERMINADO EN BASE A LA PROYECCIN DE VARIACIONES DE CONSUMO Y EL CAUDAL DE DISEO PARA EL AO 2031, USANDO P
Q dis( 2026) =

0.00 litros/seg.

ones de consumo del ao 1999 para las zonas A y B

GASTO

Factor a =

100% demanda.2008

PROM.(2031)
@ 2 horas
% del Qprom.

DE LA LOCAL.

(Lts.)

(Lt./seg.)

(Lts./s.)

VOLUM. DE

HORAS

CONS.RESERVORIO

718,326
1,336,050
1,484,298
827,730
104,094
-619,542
-1,376,712
-1,664,406
-2,119,770
-2,977,542
-3,432,906
-3,049,920
-3,049,920

60.00%
65.00%
70.00%
85.00%
120.00%
115.00%
145.00%
125.00%
110.00%
115.00%
100.00%
90.00%
60.00%

4,917,204

4,917.2

GASTO

VOLUM.

PROM @ 2HR.

AUDAL DE 151 A 186.3, Qd= 35.3 Lt./seg.


O GENERADA AL 2009

(Lts.)

PRODUCC. PROY.

VOLUM. DE

ACUMUL.

ACUM.24Hr.2008

CONS.RESERVORIO

CONSUMO (2008)

(C / 151Lit/seg.)

100% proy.demanda.2015

(Lts.)

(Lts.)

(Lts.)
0

54.00%

0.000

2-4

58.50%

0.000

4-6

63.00%

0.000

6-8

76.50%

0.000

8-10

108.00%

0.000

10-12

103.50%

0.000

12-14

130.50%

0.000

14-16

112.50%

0.000

16-18

99.00%

0.000

18-20

103.50%

0.000

20-22

90.00%

0.000

22-24

81.00%

0.000

0-2

54.00%

0.000

0.00

Reserv(1003.28 l/s.)=

0.00
(Lts./s.)

VOLUMEN

0-2

POR DA

Vol.diseo ampliac.reserv.2008

0.9

LOCALIDAD (Consumo prom.usado en el diseo del proyecto(2000) con variaciones de consumo del ao 1999 para
las zonas A y B Pd=15 aos (2015) )
CUADRO 03.-

0
(Lts.)

0.00

(M3)

Vol.diseo inic.reservorio 2026:

0.00

Se calcula con ste dato (2031) por encontarse cercano al ao del periodo de diseo calculado de 15 aos (2012 + 15 =2027)

EL CAUDAL DE DISEO PARA EL AO 2031, USANDO PD=20 Aos (2012-2031)

RELACIN DIAMETRO - VELOCIDAD ECONMICA

diseo del proyecto(2000) con variaciones de consumo del ao 1999 para

s A y B Pd=15 aos (2015)

(Ref. : NORMAS INOS-VENEZOLANAS)


DIMETRO

M3

x 1.3 =

diseo calculado de 15 aos (2012 + 15 =2027)

0.00 M3

mm.

Pulg.

V mx. (m/seg.)

Q (m3/seg.)
3.05

75

3"

0.70

100

4"

0.75

5.89

150

6"

0.80

14.14

200

8"

0.90

28.27

250

10"

1.00

49.09

300

12"

1.10

77.75

350

14"

1.20

115.45

400

16"

1.25

157.10

450

18"

1.30

206.76

500

20"

1.40

274.90

600

24"

1.60

452.39

750

30"

1.60

729.60

FACULTAD DE INGENIERIA CIVIL Y ARQUITECTURA


ESCUELA ACDEMICA PROFESIONAL DE INGENIERA CIVIL

SOLUCIONARIO DEL EXAMEN DE SUBSANACIN(CICLO 2.2009 (14/03/10) )


RESP.DEL REA HIDRAULICA :
ASIGNATURA

Ing. Civil Clifton Paucar y Montenegro - REG. CIP. 45773


ABASTECIMIENTOS DE AGUA Y ALCANTARILLADO - CIC404

HOJA N 01/09

RPTA A.-

i)

Iniciamos el proceso de estimacin calculando la dotacin promedio CATASTRAL, en funcin al consumo


promedio cuantificado por tipo de EDIFICACIN Y HABITANTE, as como el consumo promedio en Lt./seg.
80,500

Poblacin total localidad ALFA

Habitantes

ZONA A :

Densidad poblacional promedio :

DATOS

TIPO1( unid.)=

1,000

TIPO2( unid.) =

885

TIPO3( unid.) =

500

TIPO4( unid.) =

300

Lit./viv./da

Lit./hab./da

Lit./viv./da

Lit./hab./da

Lit./viv./da

Lit./hab./da

Lit./viv./da

Lit./hab./da

430
345
380

2,230
1,790
2,020

370
295
325

1,400
1,200
1,302

230
188
206

1,250
940
1,120

206
155
195

Mximo
2,600
Mnimo
2,100
Promedio
2,300
TOTAL VIVIENDAS

2,685

Consumo promedio por edificacin/ da =

Poblacin en el Ao 2009:

(RED EXISTENTE)

6.00 habitantes por vivienda

1,000 x 2,300 885 x 2,020 500 x 1,250 300 x 1,118


(1,000865 500300 )

20.00%

80,500 Hab.

Densidad poblacional promedio por edificacin


Dotacin para el AO 2009 en base al consumo por tipo de edificacin

1,890.02 Lit./edif./da

16,100 hab.

6.00 hab

315.00 Lt/hab./da

(x C.R:Includo 17% de prdidas y desperdicios en la red de reciente instalacin)

Dotacin para el AO 2009 en base al consumo por habitante

308.80 Lt/hab./da

(xC.C:Incidencia futura de incremento de consumo por la tendencia de crecimiento comercial)

TOMANDO EL VALOR MAYOR DE DOTACIN TENDREMOS


Consumo promedio instantaneo para la zona A

58.70 Lt./seg.

(RED EXISTENTE)

ZONA B :

Densidad poblacional promedio :

DATOS

TIPO1( unid.)=

2000

TIPO2( unid.) =

1500

TIPO3( unid.) =

800

TIPO4( unid.) =

620

Lit./viv./da

Lit./hab./da

Lit./viv./da

Lit./hab./da

Lit./viv./da

Lit./hab./da

Lit./viv./da

Lit./hab./da

240
190
200

1,810
1,450
1,648

200
160
180

1,750
1,220
1,370

190
135
150

1,400
1,040
1,141

150
115
128

Mximo
2,200
Mnimo
1,750
Promedio
1,850
TOTAL VIVIENDAS

4,920

Consumo promedio por edificacin/ da =


Poblacin en el Ao 2009:

9.00 habitantes por vivienda

2,000 x1,850 1,500 x 1,648 800x 1,370 620x 1,141


( 2,0001,500 800 620 )

55.00%

80,500 Hab.

Densidad poblacional promedio por edificacin


Dotacin para el AO 2009 en base al consumo por tipo de edificacin

1,621.02 Lit./edif./da

44,275 hab.

9.00 hab

180.11 Lt/hab./da

(x C.R:Includo 45% de prdidas y desperdicios en la red de instalacin antigua)

Dotacin para el AO 2009 en base al consumo por habitante

176.70 Lt/hab./da

(xC.C:Incidencia futura de incremento de consumo al mantener la tendencia de crecimiento sostenido de tipo residencial)

TOMANDO EL VALOR MAYOR DE DOTACIN TENDREMOS


Consumo promedio instantaneo para la zona B

92.30 Lt./seg.

(RED EN PROYECCIN)

ZONA C :

Densidad poblacional promedio :

DATOS

TIPO1( unid.)=

1300

TIPO2( unid.) =

1800

TIPO3( unid.) =

1200

TIPO4( unid.) =

730

Lit./viv./da

Lit./hab./da

Lit./viv./da

Lit./hab./da

Lit./viv./da

Lit./hab./da

Lit./viv./da

Lit./hab./da

700
600
650

200
165
180

700
550
600

190
150
165

700
450
500

180
125
132

550
350
350

140
90
100

Mximo
Mnimo
Promedio

4.00 habitantes por vivienda

FACULTAD DE INGENIERIA CIVIL Y ARQUITECTURA


ESCUELA ACDEMICA PROFESIONAL DE INGENIERA CIVIL

SOLUCIONARIO DEL EXAMEN DE SUBSANACIN(CICLO 2.2009 (14/03/10) )


RESP.DEL REA HIDRAULICA :

Ing. Civil Clifton Paucar y Montenegro - REG. CIP. 45773

ABASTECIMIENTOS DE AGUA Y ALCANTARILLADO - CIC404

ASIGNATURA

Consumo promedio por edificacin/ da =

Poblacin en el Ao 2009:

1,300 x 650 1,800 x 600 1,200 x 500 730 x 350


(1,300 1,800 1,200 730 )

25.00%

80,500 Hab.

Densidad poblacional promedio por edificacin


Dotacin para el AO 2009 en base al consumo por tipo de edificacin

HOJA N 02/09

552.78 Lit./edif./da

20,125 hab.

4.00 hab

138.20 Lt/hab./da

(x C.R:Includo 17% de prdidas y desperdicios en la red de reciente instalacin)

Dotacin para el AO 2009 en base al consumo por habitante

151.57 Lt/hab./da

(xC.C:Incidencia futura de incremento de consumo por la tendencia de crecimiento comercial)

TOMANDO EL VALOR MAYOR DE DOTACIN TENDREMOS


Consumo promedio instantaneo para la zona C

35.30 Lt./seg.

Rpta. A y B
ZONA CON SERVICIO DE AGUA

ZONA SIN SERVICIO DE AGUA

CUADRO DE RESPUESTAS A Y B.DEMANDA CATASTRAL PROMEDIO:

Zona "A"

Zona "B"

Zona "C"

Densidad prom.poblac.(hab./edif.)

6.00

9.00

4.00

Poblacin Total ao 2009 ( habitantes )


Dotacin para el ao 2009 (Lt./hab./da)

16,100
315.00

44,275
180.11

20,125
151.57

80,500.00

Consumo prom.ao 2009 ( Lt./seg. )

58.70

92.30

35.30

186.30

Q de diseo del sistema ao 2000(Dato)

Total

DEFICIT./SUPERAV.

123.4%

151.00

Rpta. C
CUADRO DE RESPUESTA PARA EL INCISO C.DEMANDA CATASTRAL POR EDIFICACIN
Includo/PRDIDAS:

Zona "A"

Zona "B"

Zona "C"

6.00

9.00

4.00

Poblacin Total ao 2009 ( habitantes )

16,100

44,275

20,125

Dotacin para el ao 2009 calculada por


edificacin sin prdidas(Lt./hab./da)

315.00

180.11

138.20

Dotacin para el ao 2009 Inc. Prdidas segn datos


suministrados 60%-45%-17% (Lt./hab./da)
Dotacin para el ao 2015 Inc. Prdidas segn datos
proyectados 60%-45%-17% (Lt./hab./da)
Dotacin para el ao 2021 Inc. Prdidas segn datos
proyectados 60%-45%-17% (Lt./hab./da)
Dotacin para el ao 2031 Inc. Prdidas segn datos
proyectados 60%-45%-17% (Lt./hab./da)

504.00

261.16

161.69

504.00

261.16

161.69

504.00

261.16

161.69

504.00

261.16

161.69

Consumo prom.ao 2009 sin / Prdidas


(Lt./seg. )
Consumo prom.ao 2009 includo/ Prdidas
(Lt./seg. )

58.70

92.30

32.19

183.19

93.92

133.83

37.66

265.41

Poblacin proyectada ao 2015

0.00

0.00

0.00

0.00

Consumo prom.ao 2015 sin / Prdidas


(Lt./seg. )
Consumo prom.ao 2015 Includo / Prdidas
(Lt./seg. )

0.00

0.00

0.00

0.00

0.00

0.00

0.00

0.00

Poblacin proyectada ao 2021

0.00

0.00

0.00

0.00

Consumo prom.ao 2021 sin / Prdidas


(Lt./seg. )
Consumo prom.ao 2021 Includo / Prdidas
(Lt./seg. )

0.00

0.00

0.00

0.00

0.00

0.00

0.00

0.00

Poblacin proyectada ao 2031

0.00

0.00

0.00

0.00

Consumo prom.ao 2031 sin / Prdidas


(Lt./seg. )
Consumo prom.ao 2031 Includo / Prdidas
(Lt./seg. )

0.00

0.00

0.00

0.00

0.00

0.00

0.00

0.00

#DIV/0!

PROM.:

#DIV/0!

Densidad prom.poblac.(hab./edif.)

Q de diseo del sistema ao 2000(Dato)

151.00

Total

80,500.00

144.9%

#DIV/0!

#DIV/0!

EL ANLISIS DE LA VARIACIN DE CONSUMO EN EL TIEMPO SE EFECTUAR CON LA DECISIN DE USAR PARA EL CALCULO DE LA
DOTACIN INCREMENTADA , UNA ECUACIN LINEAL CON LOS DATOS (ZONAS URBANAS-DONDE EL INCREMENTO DEL CONSUMO NO
SOLO DEPENDE DEL CRECIMIENTO POBLACIONAL) O EL MODELO MATEMATICO DE CAPEN (ZONAS RURALES-DONDE LA DOTACIN
SE BASA EN EL USO DOMESTICO Y SU VARIACIN DEPENDE DEL INCREMENTO POBLACIONAL), PARA LO CUL ANTES DEBEMOS
ESTIMAR EL QPROMEDIO DE DISEO SEGN LAS CARACTERISTICAS DE CONSUMO DIARIO Y HORARIO, LOS FACTORES DE
MAYORACIN K1 Y K2 Y LA POBLACIN FUTURA

ii)

Efectuamos las estimaciones de las demanda por variaciones de consumo diarias en funcin a los datos
suministrados del diario promedio horario y del diario maximo horario en Lt./seg., a fin de determinar los
valores de K2.

Para determinar el caudal medio en Lt./seg. durante el ao 2009, se proceder a desarrollar el cuadro que contiene las variaciones horarias del
consumo promedio durante el ao, que representa en forma analitica a la curva masa (Produccin y Demanda vrs. Tiempo), teniendo como base que
se consume 151.00 Lt./seg. (2009), y que es la produccin total de la captacin (100%) que es de 151 Lt.seg. con el cul se diseo el reservorio en el
ao 2000, proyecto que entr en funcionamiento el ao 2001 con un Pd=15 aos, debiendo cumplir con eficiencia la demanda hasta el ao 2015,
PERO SE OBSERVA EN EL CUADRO 01 que slo cumple con la demanda del ao 2009 PARA LAS ZONA A Y B, Y QUE DE INCLUIRSE LA
DEMANDA INSATISFECHA EN C QUE NO CUENTA CON SISTEMA, EXISTE UN DEFICIT EN ESTE AO 2009 DE 44.4% .

DATO:

HORAS

Qdisp. Mx.=

1,001 Litros/seg.

LOCALIDAD ( Consumo promedio ao 2009 )

GASTO

CUADRO 02.-

PROM.
@ 2 horas
% del Qprom.

GASTO

VOLUM.(2009)

PRODUCC.(2009)

VOLUM. DE

SIMULACIN

VOLUM. DE

PROM @ 2HR.

VOLUMEN

ACUM.CONSUMO

ACUM.24Hr.

CONS.RESERV.

PRODUCC.(2009)

CONS.RESERV.

(186.30 Lt./seg.)

(186.30 Lt./seg.)

(186.30 Lt./seg.)

(151 Lt./seg.)

(Demanda 186.3)

(186.30 Lt./seg.)

(Demanda 186.30 Lt./s)

(Lt./seg.)

(Lts./s.)

(Lts.)

(Lts.)

(Lts.)

(Lts.)

(Lts.)

(Lts.)

25.00%

46.575

368,874

368,874

1,087,200

718,326

1,341,360

972,486

30.00%

55.890

469,476

838,350

2,174,400

1,336,050

2,682,720

1,844,370

40.00%

74.520

938,952

1,777,302

3,261,600

1,484,298

4,024,080

2,246,778

100.00%

186.300

1,743,768

3,521,070

4,348,800

827,730

5,365,440

1,844,370

160.00%

298.080

1,810,836

5,331,906

5,436,000

104,094

6,706,800

1,374,894

10

110.00%

204.930

1,810,836

7,142,742

6,523,200

-619,542

8,048,160

905,418

12

160.00%

298.080

1,844,370

8,987,112

7,610,400

-1,376,712

9,389,520

402,408

14

115.00%

214.245

1,374,894

10,362,006

8,697,600

-1,664,406

10,730,880

368,874

16

90.00%

167.670

1,542,564

11,904,570

9,784,800

-2,119,770

12,072,240

167,670

18

140.00%

260.820

1,944,972

13,849,542

10,872,000

-2,977,542

13,413,600

-435,942

20

150.00%

279.450

1,542,564

15,392,106

11,959,200

-3,432,906

14,754,960

-637,146

22

80.00%

149.040

704,214

16,096,320

13,046,400

-3,049,920

16,096,320

24

25.00%

46.575

16,096,320

13,046,400

POR DA

2,235.60
(Lts./s.)

16,096,320

16,096.32

(Lts.)

(M3)

186.30

k2prom.09:

Reserv.151 l/s.

16,096,320
4,917,204

Vol.diseo inic.reservorio 95:

Reserv(186.3 l/s.)=

1,441,962

Vol.diseo ampliac.reservor 2008

1.60

VALOR SIN APLICACIN DEL FACTOR DE MAYORACION K1 =1.3


4,917.2
M3
1,442.0
Como se tiene como dato que la persistencia de consumo de los valores mximos diarios es de 10 a 13% se utilizar el cuadro No. 02 para estimar el
valor de K2 y el Volumen de consumo del reservorio principal de ampliacin. Se muestra los graficos de las curvas masa en ambos casos con fines
de comparacin de los consumos.

FACULTAD DE INGENIERIA CIVIL Y ARQUITECTURA


ESCUELA ACDEMICA PROFESIONAL DE INGENIERA CIVIL

SOLUCIONARIO DEL EXAMEN DE SUBSANACIN(CICLO 2.2009 (14/03/10) )


RESP.DEL REA HIDRAULICA :
ASIGNATURA

50000000
45000000
40000000

35000000
30000000
25000000

Ing. Civil Clifton Paucar y Montenegro - REG. CIP. 45773


ABASTECIMIENTOS DE AGUA Y ALCANTARILLADO - CIC404

HOJA N 03/09

25000000
20000000

15000000
10000000
5000000
0

10

12

14

16

18

20

22

SIMULACIN PRODUCCion.(2009) (54.38 Lt./seg.) (Lts.)


PRODUCC.(2009) ACUM.24Hr. (551 Lt./seg.) (Lts.)

SIMULACIN PRODUCCion.(2009) (54.38 Lt./seg.) (Lts.)


PRODUCC.(2009) ACUM.24Hr. (186.3 Lt./seg.) (Lts.)

VOLUM. ACUMUL. Cons.da mx. (Lts.)

HORAS

GASTO
CUADRO 03.PROM.
GASTO
@ 2 horas
PROM @ 2HR.
% del Qprom.
(186.30 Lt./seg.)
(Lt./seg.)

(Lts./s.)

LOCALIDAD (Consumo del da mximo ao 2009 )


VOLUM.

PRODUCC.(2009)

VOLUM. DE

VOLUMEN

ACUMUL.

ACUM.24Hr.

CONSUM.RES. (100%)

(186.30 Lt./seg.)

Cons.da mx.

(186.3 Lt./seg.)

Cons.da mx.

(Lts.)

(Lts.)

(Lts.)

(Lts.)

30.00%

55.89

435,960

435,960

1,087,200

651,240

35.00%

65.21

536,580

972,540

2,174,400

1,201,860
1,283,004

45.00%

83.84

1,006,056

1,978,596

3,261,600

105.00%

195.62

1,810,872

3,789,468

4,348,800

559,332

165.00%

307.40

1,911,456

5,700,924

5,436,000

-264,924

10

120.00%

223.56

2,146,176

7,847,100

6,523,200

-1,323,900

12

200.00%

372.60

2,112,660

9,959,760

7,610,400

-2,349,360

14

115.00%

214.25

1,408,464

11,368,224

8,697,600

-2,670,624

16

95.00%

176.99

1,609,668

12,977,892

9,784,800

-3,193,092

18

145.00%

270.14

2,045,592

15,023,484

10,872,000

-4,151,484

20

160.00%

298.08

1,643,184

16,666,668

11,959,200

-4,707,468

22

85.00%

158.36

771,300

17,437,968

13,046,400

-4,391,568

24

30.00%

55.89

17,437,968

13,046,400

-4,391,568

17,437.97

Reserv(186.3 l./s.)=

5,990,472

POR DA

2,421.94
(Lts./s.)

Rpta. D:

k1=

1.30

17,437,968
(Lts.)

(M3)

Vol.diseo ampliac.reservor 2008 por cons.mx.horar.

201.83

k2dis.(2008):

2.00 (Coef.da de mx.consumo)

(Reglam.)

k2dis.09:

1.60 (Consumo Promedio)

5,990.00

M3

Yi 15.77 e

0.00005

Xi
CURVA DE CRECIMIENTO POBLACIONAL ZONA B

20,000

18,000

POBLACIN

16,000

14,000

12,000

10,000

8,000

6,000

4,000

2,000

0
2004 2009 2014 2019 2024 2029 2034 2039 2044 2049 2054 2059 2064 2069 2074

AO

POBLACIN

Yi 25.66 0.00015 X i

TASAS CREC. POB.

CURVA DE CRECIMIENTO POBLACIONAL ZONA A


1

POBLACIN

0
1

AO

Yi 77.17 e

0.00004

Xi

CURVA DE CRECIMIENTO POBLACIONAL ZONA A

CURVA DE CRECIMIENTO POBLACIONAL ZONA A


140,000

120,000

POBLACIN

100,000

80,000

60,000

40,000

20,000

0
200420092014201920242029203420392044

AO

POBLACIN

TASAS CREC. POB.

1 Pto.
1 Pto.

1 Pto.

1 Pto.

SE USARA LA VARIACION LINEAL POR


QUE LA DIFERENCIA ES SIGNIFICATIVA
CON LA PROYECCION USANDO EL
MODELO MATEMATICO DE CAPEN

Q diseo = 1.3 X

186.30

242.19

VOLUM.

PRODUCC.PROY.x1.3

VOLUM. DE

Lit./seg.

ACUMUL. X 1.3
CONSUMO (2009)

ACUM.24Hr.2009
(C /242.19 Lit/seg.)

(Lts.)

(Lts.)

Caudal de diseo de la tubera de entrada del Reservorio

CONS.RESERVORIO

Usando la tabla que seala velocidades mximas y gastos mximos admisibles para
cada dimetro, propuesta por las normas Venezolanas (INOS-Ref.3)

Mayorado c/ K1=1,3

(Lts.)

=
479,536

1,413,360

933,824

1,089,855

2,826,720

1,736,865

2,310,493

4,240,080

1,929,587

4,577,391

5,653,440

1,076,049

6,931,478

7,066,800

135,322

9,285,565

8,480,160

-805,405

11,683,246

9,893,520

-1,789,726

13,470,608

11,306,880

-2,163,728

15,475,941

12,720,240

-2,755,701

18,004,405

14,133,600

-3,870,805

20,009,738

15,546,960

-4,462,778

20,925,216

16,960,320

-3,964,896

20,925,216

16,960,320

4,917 X1.3

20"
REQUERIMIENTO DE DISEO (2009)
186.30 x 1.3 =
242.19 l/s
20"

ANTERIOR DISEO (2000)

151.00 l/s
16"

-3,964,896

6,392,365
6,392.4 M3

6,392.4

1,442.0 X 1.3 =

1,875 M3

Diferencia generada por la produccin, que se entiende fsicamente que si no se cambia el dimetro de la tubera de acceso al Reservorio se tendra que
construir un RESERVORIO de (4,917.2-1,442.0=3,475.20 m3), considerando que en el ao 2000 se uso los mismos % de variaciones de consumo del 2009; si se
considera el cambio de tubera de acceso para transportar 186.30 Lt./s. solo se necesitara u Reservorio de 1, 442 m3 que deber contrastarse con el volumen de
consumo del reservorio construdo ( Evaluacin de costos y ventajas tcnicas)

-3,475.2 M3
DEFICIT DE SERVICIO EN EL AO 2009 AL PRODUCIR 151 Lit./ seg. PARA CUBRIR UNA DEMANDA DE 186.30 Lit./s. NETA DE
CONSUMO POR EDIFICACIN SIN CONSIDERAR PERDIDAS+CONS.PBLICO+OPERAC.YMANTEN.VARIAC. DE DOTACIN
CON EL TIEMPO.

DECIDIMOS INCREMENTAR E
POR LO QUE NECESITAMOS
NECESITAMOS CALCULAR LA

M3

2 Ptos.

VARIA NOTORIAMENTE AL INCREMENTARSE LOS % DE VARIACIONDE CONSUMO MANTENIENDOSE EL Q=186.3 Lit/seg. Cte. como
1,442.0
5,990.00
Q demanda mxima horaria = 20
<
EL VOLUMEN DEL RESERVORIO EN EL 2031 SERA:
1,442.0

1.3

1,874.6 M3

CONSUMO DETERMINADO EN EL EXPEDIENTE DEL PROYECTO PARA DETERMINAR EL VOLUMEN DEL RESERVORIO EXISTENTE ( 2000) CON PD=15 Aos

GASTO
HORAS

CUADRO 03.-

PROM.(1999)
@ 2 horas
% del Qprom.

DE LA LOCAL.

(Lt./seg.)

(Lts./s.)

LOCALIDAD (Consumo prom.usado en el diseo del proyecto(2000) con variaciones de consumo del ao 1999 para las zonas A y B
Pd=15 aos (2015))

GASTO
PROM @ 2HR.

VOLUM.
VOLUMEN

(Lts.)

PRODUCC. PROY.

VOLUM. DE

CONSUMO REAL

ACUMUL.

ACUM.24Hr.2015

CONS.RESERVORIO

ACUM.24Hr.2009

CONSUMO (2015)

(C / 151Lit/seg.)

100% proy.demanda.2015

(C / 186.30 Lit/seg.)

(Lts.)

(Lts.)

(Lts.)

(Lts.)

0-2

5.00%

7.550

108,720

108,720

1,087,200

978,480

2-4

15.00%

22.650

217,440

326,160

2,174,400

1,848,240

368,874
838,350

4-6

25.00%

37.750

597,960

924,120

3,261,600

2,337,480

1,777,302

6-8

85.00%

128.350

1,304,640

2,228,760

4,348,800

2,120,040

3,521,070

8-10

155.00%

234.050

1,522,080

3,750,840

5,436,000

1,685,160

5,331,906

10-12

125.00%

188.750

2,147,220

5,898,060

6,523,200

625,140

7,142,742

12-14

270.00%

407.700

2,283,120

8,181,180

7,610,400

-570,780

8,987,112

14-16

150.00%

226.500

1,386,180

9,567,360

8,697,600

-869,760

10,362,006

16-18

105.00%

158.550

1,359,000

10,926,360

9,784,800

-1,141,560

11,904,570

18-20

145.00%

218.950

1,168,740

12,095,100

10,872,000

-1,223,100

13,849,542

20-22

70.00%

105.700

652,320

12,747,420

11,959,200

-788,220

15,392,106

22-24

50.00%

75.500

298,980

13,046,400

13,046,400

16,096,320

0-2

5.00%

7.550

13,046,400

13,046,400

13,046.40

Reserv(343.68 l/s.)=

1,114,380

Reserv(343.68 l/s.)=

POR DA

1,812.00
(Lts./s.)

13,046,400
(Lts.)

(M3)

151.00

Volumen de consumo del Reservorio si se utiliza los % de variacin del 2009


DIFERENCIA

16,096,320

Vol.diseo inic.reservorio 95:

Vol.diseo ampliac.reserv.2008

1,114.0

M3

1,442.0
328.0

M3
M3

DECIDIMOS INCREMENTAR EN FORMA PARALELA UNA LINEA DE CONDUCCIN PARA INCREMENTAR EL CAUDAL DE 151 A 186.3, Qd= 35.3 Lt./seg.
POR LO QUE NECESITAMOS INCREMENTAR UN RESERVORIO DE 328 M3 PARA CUBRIR LA NECESIDAD SOLO GENERADA AL 2009
NECESITAMOS CALCULAR LA NECESIDAD PARA EL AO 2031 CONSIDERANDO UN PERIODO DE DISEO INICIAL DE 20 AOS

NDOSE EL Q=186.3 Lit/seg. Cte. como produccin


Q demanda mxima horaria = 201.83

ISTENTE ( 2000) CON PD=15 Aos

CONSUMO DETERMINADO EN BASE A LA PROYECCIN DE VARIACIONES DE CONSUMO Y EL CAUDAL DE DISEO PARA EL AO 2031, USANDO P
Q dis( 2026) =

0.00 litros/seg.

ones de consumo del ao 1999 para las zonas A y B

GASTO

Factor a =

100% demanda.2008

PROM.(2031)
@ 2 horas
% del Qprom.

DE LA LOCAL.

(Lts.)

(Lt./seg.)

(Lts./s.)

VOLUM. DE

HORAS

CONS.RESERVORIO

718,326
1,336,050
1,484,298
827,730
104,094
-619,542
-1,376,712
-1,664,406
-2,119,770
-2,977,542
-3,432,906
-3,049,920
-3,049,920

60.00%
65.00%
70.00%
85.00%
120.00%
115.00%
145.00%
125.00%
110.00%
115.00%
100.00%
90.00%
60.00%

4,917,204

4,917.2

GASTO

VOLUM.

PROM @ 2HR.

AUDAL DE 151 A 186.3, Qd= 35.3 Lt./seg.


O GENERADA AL 2009

(Lts.)

PRODUCC. PROY.

VOLUM. DE

ACUMUL.

ACUM.24Hr.2008

CONS.RESERVORIO

CONSUMO (2008)

(C / 151Lit/seg.)

100% proy.demanda.2015

(Lts.)

(Lts.)

(Lts.)
0

51.00%

0.000

2-4

55.25%

0.000

4-6

59.50%

0.000

6-8

72.25%

0.000

8-10

102.00%

0.000

10-12

97.75%

0.000

12-14

123.25%

0.000

14-16

106.25%

0.000

16-18

93.50%

0.000

18-20

97.75%

0.000

20-22

85.00%

0.000

22-24

76.50%

0.000

0-2

51.00%

0.000

0.00

Reserv(1003.28 l/s.)=

0.00
(Lts./s.)

VOLUMEN

0-2

POR DA

Vol.diseo ampliac.reserv.2008

0.85

LOCALIDAD (Consumo prom.usado en el diseo del proyecto(2000) con variaciones de consumo del ao 1999 para
las zonas A y B Pd=15 aos (2015) )
CUADRO 03.-

0
(Lts.)

0.00

(M3)

Vol.diseo inic.reservorio 2026:

0.00

Se calcula con ste dato (2031) por encontarse cercano al ao del periodo de diseo calculado de 15 aos (2012 + 15 =2027)

EL CAUDAL DE DISEO PARA EL AO 2031, USANDO PD=20 Aos (2012-2031)

RELACIN DIAMETRO - VELOCIDAD ECONMICA

diseo del proyecto(2000) con variaciones de consumo del ao 1999 para

s A y B Pd=15 aos (2015)

(Ref. : NORMAS INOS-VENEZOLANAS)


DIMETRO

M3

x 1.3 =

diseo calculado de 15 aos (2012 + 15 =2027)

0.00 M3

mm.

Pulg.

V mx. (m/seg.)

Q (m3/seg.)
3.05

75

3"

0.70

100

4"

0.75

5.89

150

6"

0.80

14.14

200

8"

0.90

28.27

250

10"

1.00

49.09

300

12"

1.10

77.75

350

14"

1.20

115.45

400

16"

1.25

157.10

450

18"

1.30

206.76

500

20"

1.40

274.90

600

24"

1.60

452.39

750

30"

1.60

729.60

También podría gustarte